9/6-MBE REAL PROPERTY

Pataasin ang iyong marka sa homework at exams ngayon gamit ang Quizwiz!

Question 5874 A speculator and the original owner of a condominium unit entered into a contract for the sale of the unit. The contract, which contained no reference to the marketability of the title, called for the owner to transfer the unit to the speculator by quitclaim deed, which the owner did on the date called for in the contract. A year later, the speculator entered into a contract to sell the unit to a third party at a price significantly higher than the price paid by the speculator for the unit. The contract specifically required the speculator to provide the third party with title to the unit free from all defects. Upon investigation, the third party discovered that the unit was subject to a restrictive covenant that rendered the title to the unit unmarketable, and that the restrictive covenant had existed at the time that the speculator had purchased the unit. The third party refused to complete the transaction. The speculator subsequently sued the original owner of the condominium unit for breach of contract. For whom is the court likely to rule? Answers: The speculator, because a covenant of marketable title was implied in the contract. The speculator, because of the warranty against encumbrances. The original owner, because the condominium unit was transferred by a quitclaim deed. The original owner, because of the merger doctrine.

Answer choice D is correct. Absent contrary language, an implied covenant of marketable title (i.e., a title free from defects) is part of a contract to sell real property. However, under the doctrine of merger, obligations contained in the contract of sale, including the seller's duty to deliver marketable title, are merged into the deed and cannot thereafter be enforced through a breach of contract action. Consequently, the speculator cannot sue the original owner for a breach of the covenant to deliver marketable title. Answer choice A is incorrect. Although there is an implied covenant of marketable title in a contract to sell real property, this covenant merges into the deed and cannot thereafter be enforced through a breach of contract action. Answer choice B is incorrect because, although there is an implied covenant of marketable title in a contract to sell real property, this covenant merges into the deed and cannot thereafter be enforced through a breach of contract action. Moreover, because the original owner transferred the condo unit by a quitclaim deed, the original owner did not give the speculator any general title warranties, include the warranty against encumbrances. Answer choice C is incorrect. The contract between the original owner and the speculator is treated as containing a covenant of marketability even though the contract did not specifically provide for this covenant. This covenant is implied even in the case of a contract that calls for transfer by a quitclaim deed. However, because of the doctrine of merger, the speculator cannot now sue on the breach of this covenant.

Question 6623 A private company managed a publicly funded experimental project for the conversion of garbage into energy through biological means. Despite locating the project in a sparsely populated area on land purchased from a real-estate investment trust, the company has nevertheless received complaints from those living near the project about the repugnant smells emanating from the site. One neighbor who rents a farmhouse for use as his personal residence on an annual basis from an individual farmer can no longer reside on the property. He has filed suit against the company, seeking damages. Of the following doctrines, which provides the neighbor with the best chance for recovery? Answers: Private nuisance Public nuisance Constructive eviction Waste

Answer choice A is correct. A private nuisance is a substantial, unreasonable interference with another individual's use or enjoyment of his property. The interference may be intentional, negligent, reckless, or the result of abnormally dangerous conduct. Anyone with possessory rights in the property may bring a nuisance claim. Here, the smells emanating from the project prevent the neighbor from living in the farmhouse. Although this interference is clearly substantial, it is arguable as to whether it is unreasonable. Nevertheless, of the four options, a private nuisance provides the neighbor with the best chance of recovery. Answer choice B is incorrect. A public nuisance is an unreasonable interference with the health, safety, or property rights of the community. Although the smells emanating from the project site are adversely affecting those who live in the area, to recover for a public nuisance, a plaintiff must show that he suffered a different kind of harm than that suffered by the rest of the community. There is no indication that the harm suffered by the neighbor is different in kind from that suffered by the rest of the community in this case. Answer choice C is incorrect because constructive eviction is based on a landlord's substantial interference with the tenant's use and enjoyment of the leasehold premises by breaching a duty owed by the landlord to the tenant. Here, there is no landlord-tenant relationship between the private company that is managing the experimental project and the lessee of the neighboring farmhouse. The lessor of the farmhouse is an individual farmer. The conduct of the individual farmer has not caused the neighbor's issue. Answer choice D is incorrect. Although the neighbor is complaining of the adverse effects of the handling of garbage, a legal action for waste is available only to a plaintiff who holds an interest in the same property as the defendant, such as a life tenant and a remainderman or a tenant-in-common against another tenant-in-common. Because the neighbor does not have a property interest in the site where the project is being conducted and the private company does not have a property interest in the farmhouse rented by the neighbor, waste is not available as a cause of action that the neighbor can pursue against the company.

Question 5871 A developer obtained a loan from a bank to construct an apartment building. The loan was evidenced by a note and secured by a mortgage on the apartment building. After making the required installment payments on the note to the bank for several years, the developer defaulted on the loan. The bank elected not to foreclose on the mortgage, but instead, sued the developer personally for the unpaid balance of the note because the note contained an acceleration clause that made the entire unpaid balance due upon default. The court rendered a judgment in the bank's favor. The bank promptly and properly filed the judgment creating a lien, pursuant to the applicable state law, on any real property then owned or subsequently acquired by the developer for the next 10 years. Subsequently, the developer secured a loan from a private investor to purchase a small strip mall. This loan was also evidenced by a note and secured by a mortgage on the mall. Once again, after making the required installment payments on the note to the private investor for several years, the developer defaulted on this note. The private investor filed an action to foreclose on the mortgage it held on the mall. The bank was made a party to this action and sought to enforce its judgment lien against the mall. The court recognized the rights of both the private investor and the bank in the mall, and ordered the sale of the mall. The proceeds from the sale, after the costs associated with the sale were paid, were more than sufficient to satisfy the developer's outstanding obligation to the private investor or judgment obtained by the bank, but not both. Who has priority to the net sale proceeds? Answers: The private investor, because the funds from the loan were used to purchase the mall. The private investor, because the investor initiated the foreclosure on the mall. The bank, because the judgment lien occurred before the mortgage on the mall. The bank, because a judgment lien has priority over a mortgage.

Answer choice A is correct. A purchase-money mortgage is a mortgage granted to (i) the seller of real property or (ii) a third-party lender, to the extent that the loan proceeds are used to acquire title to the real property or construct improvements on the real property if the mortgage is given as part of the same transaction in which title is acquired. A purchase-money mortgage has priority over mortgages and liens created by or that arose against the purchaser-mortgagor prior to the purchaser mortgagor's acquisition of the property, whether or not recorded. Here, with regard to the mall, the private investor has a purchase money mortgage because the developer used the funds loaned by the private investor to purchase the mall. Consequently, the private investor's mortgage has priority over any other mortgages or liens on the mall. Answer choice B is incorrect. Although the private investor initiated the foreclosure action, the private investor's priority to the net proceeds from the sale of the mall stems from the status of the investor's mortgage as a purchase money mortgage, not from the private investor's role as plaintiff in this action. Answer choice C is incorrect because, although the judgment lien was created prior to the creation of the mortgage on the mall, the investor's mortgage on the mall has priority because it was a purchase money mortgage. Answer choice D is incorrect because a judgment lien does not automatically have priority over a mortgage.

Question 6621 Pursuant to a written lease, the owner of a warehouse leased the premises to a manufacturer for a term of one year at a total rent of $60,000. The lease called for the rent to be paid in monthly installments of $5,000 at the beginning of each month. The lease contained no provisions regarding termination or extension. The manufacturer promptly made the required rental payment each month. At the end of the year, the owner did not provide notice to the manufacturer of the termination of the lease. The manufacturer tendered a rental payment of $5,000 for the following month to the owner, which the owner refused to accept. In the absence of an applicable statute, how much advance notice must the owner give the manufacturer before seeking to evict the manufacturer? Answers: None, because the manufacturer is a tenant at sufferance. A reasonable time, because the manufacturer is a tenant at will. A month, because the manufacturer, by tendering a rental payment, has created a periodic tenancy. Six months, because the manufacturer, by tendering a rental payment, has created a tenancy for years.

Answer choice A is correct. A tenancy for years is an estate measured by a fixed and ascertainable amount of time. Termination occurs automatically upon the expiration of the term; no notice is required. Any right to renew the agreement must be explicitly set out in the lease. In this case, the owner and the manufacturer had a tenancy for years. This tenancy expired at the end of its term, which was one year. The owner was not required to give notice of its termination. Consequently, by remaining on the premises after the termination of the tenancy for years, the manufacturer became a holdover tenant (i.e., a tenant at sufferance). Because the owner refused to accept the manufacturer's tender of a rental payment, a new tenancy was not created. Therefore, the owner may file a legal action to evict the manufacturer without further notice because the manufacturer, by remaining in the warehouse past the expiration of the one-year term of the lease, is wrongfully in possession of the premises. Answer choice B is incorrect. Although a landlord must give a tenant at will a reasonable time in which to vacate the premises, the manufacturer is not a tenant at will. There was no express or implied agreement between the owner and the manufacturer for the manufacturer to continue to occupy the warehouse. Answer choice C is incorrect. Although a landlord can create a periodic tenancy by accepting a tenant's tender of a rental payment after the expiration of the lease, here the owner did not accept the manufacturer's tendered rental payment. Answer choice D is incorrect. A landlord, by accepting rent after the termination of a lease, creates a periodic tenancy, even when the prior tenancy was a tenancy for years. However, the owner did not accept the manufacturer's rental payment in this case. Thus, neither a tenancy for years nor a periodic tenancy was created. In addition, although six months' notice is required when a year-to-year periodic tenancy exists, a landlord is not required to give notice of the termination of a tenancy for years.

Question 6231 A purse maker sought to market its line of "smart" purses that were compatible with a new handheld device. As part of its plans, the purse maker sought to purchase a store from which to sell its purses. The purse maker found a suitable store, and entered into a contract with the owner of a store. The contract was in writing, signed by both parties, and stated the essential terms, including a closing date in 30 days. Due to the purse maker's plan to sell her purses in advance of the release of the new handheld device, the contract stated that the closing date could not be delayed. One week before the closing date, the purse maker discovered that the store was in violation of a zoning ordinance that mandated an updated version of the current fire sprinkler system. The owner promised that he would promptly update the fire sprinkler system and that, although it would not be finished by the closing date, it would be done in time for the grand opening of the store. In addition, the owner promised to provide a warranty deed upon closing to shield the purse maker from any potential liability stemming from the outdated fire sprinkler system. On the day before the grand opening, the purse maker refused to close the land sale deal. In an action by the owner for specific performance against the purse maker, who will prevail? Answers: The purse maker, because the owner did not provide marketable title to her on the closing date as stated in the land sales contract. The purse maker, because the warranty deed would not protect her from liability for the zoning ordinance violation. The owner, because the warranty deed would protect the purse maker from liability for the zoning ordinance violation. The owner, because the fire sprinkler system would be updated in accordance with the zoning ordinance before the store's grand opening.

Answer choice A is correct. Absent contrary language, an implied covenant of marketable title (i.e., a title free from defects) is part of a land sales contract. Here, the owner was obligated to deliver marketable title to the purse maker on the date of the closing. Generally, a court will assume that time is not of the essence in a real estate contract, unless the contract specifically states that time is of the essence, circumstances indicate that this was the intention of the parties, or one party gives the other party notice that time is of the essence. In this case, the contract specifically stated the closing date could not be delayed, and thus time was of the essence. Because the out-of-date fire sprinkler system in the warehouse was in violation of a zoning ordinance on the closing date, the title was unmarketable, and the purse maker was not obligated to close the land sale deal. This is true even if the issue was corrected within a reasonable time after the closing date because time was of the essence. Answer choice B is incorrect because the warranty deed would most likely protect the purse maker from liability for the zoning violation and, even if it did not, the owner was not entitled to specific performance of the contract because he had failed to provide marketable title to the purse maker at closing. Answer choice C is incorrect because, although the warranty deed would likely protect the purse maker from liability, the owner is still required to provide marketable title upon closing. Answer choice D is incorrect because even if the fire sprinkler system was updated before the store's grand opening, this did not obligate the purse maker to close without marketable title.

Question 6216 A mother owned a vacation cabin, but as she no longer visited it, she decided to convey the cabin to her daughter. The mother executed a valid, written deed, and promptly and properly recorded it. The mother did not tell her daughter about her intention of giving the cabin to her because she wanted to surprise her with the gift at an upcoming family reunion. Prior to the reunion, the daughter died suddenly. In her will, the daughter left her entire estate to her best friend. The mother, not wanting the cabin to go to someone who was not a family member, brought an action to set aside the conveyance to the best friend. Who will prevail? Answers: The best friend, because the mother recorded the deed conveying the cabin to her daughter. The best friend, because the mother's intent was evidenced by a valid deed in writing. The mother, because she did not deliver the deed to her daughter. The mother, because the daughter did not accept her mother's gift.

Answer choice A is correct. Although it is often stated that a deed must be delivered in order for a real property interest to pass (i.e., a delivery requirement), the term "delivery" is used as shorthand for the existence of the necessary grantor intent. Physical transfer of a deed is not required and is not conclusive evidence of the grantor's intent. The execution and recording of a deed creates a rebuttable presumption that the deed is to be presently operative. Here, there is a presumption of delivery because the mother recorded the deed. Answer choice B is incorrect because, although a valid, written deed does evidence the mother's intent to make a gift to her daughter, the deed on its own does not create a presumption of delivery. Answer choice C is incorrect because actual, physical delivery is not required--the recording of the deed was sufficient to create a presumption of delivery. Answer choice D is incorrect because, although acceptance is required for a transfer to be complete, the grantee is generally presumed to have accepted any beneficial conveyance. Acceptance relates back to the time the deed was transferred, unless a bona fide purchaser or creditor of the grantor would be negatively affected by doing so, which is not the case here.

Question 3147 The owner of undeveloped land had his attorney prepare a deed transferring the land to his niece as a surprise. The uncle signed the deed and had his attorney record it. However, before the uncle delivered the deed to his niece, the two had a falling out. After the man's death, the niece learned about the deed when it was found among her uncle's papers. She transferred her interest in the land by quitclaim deed to a good faith buyer for valuable consideration. The buyer promptly recorded this deed. By will, the owner left his real property to a nephew. The recording statute in the applicable jurisdiction states: "Any conveyance of an interest in land shall not be valid against any subsequent purchaser for value, without notice thereof, whose conveyance is recorded." Who owns the land? Answers: The nephew, because he was the devisee of the owner's real property. The nephew, because the buyer's interest in the land was obtained through a quitclaim deed. The buyer, because the uncle's deed to his niece was recorded. The buyer, because the buyer gave valuable consideration for the niece's interest.

Answer choice A is correct. Although recording the deed raises the presumption of delivery, that presumption is rebuttable by facts indicating the grantor's contrary intent. Here, the facts indicate the uncle and niece had a falling out before the uncle delivered the deed, and the deed was never delivered; this may be an indication that the uncle to change his mind, and the presumption is thereby rebutted. Because the uncle retained ownership of the land at his death, the land passed by will to his nephew. Answer choice B is incorrect because the property could have been transferred by quitclaim deed if the niece had owned it. The transfer of a real property interest through a quitclaim deed does not by itself place the transferee on notice as to a problem with the transferor's ownership of the transferred property. Instead, a quitclaim deed merely limits the liability of the transferor. Answer choice C is incorrect because, although the recording of a deed generally protects a person who buys the property from the record owner, the recording of a deed does not validate an otherwise invalid transfer. Here, since the uncle did not complete the gift to his niece by delivering the deed to her, and the presumption that a deed recorded is a deed delivered has been rebutted, she did not obtain ownership of the land and therefore could not sell it to a third party. Answer choice D is incorrect because, although a good faith purchaser of real property from the record owner is generally protected from other claimants to the property, in this case the record owner never obtained ownership of the land.

Question 5201 A wealthy individual loaned money to a friend to facilitate the purchase of a home. The friend executed a promissory note, payable to the lender, in the amount of the loan plus interest and gave the lender a mortgage on the home as security for repayment of the loan. Subsequently the friend sold the home to an unrelated third party. The documents associated with this sale made no mention of the third party's liability with respect to the loan. Later, when the value of the home exceeded the outstanding obligation on the note, the lender released her mortgage interest in the home. Shortly thereafter, the loan fell into default. The third party is the current record owner of the home. Can the lender successfully maintain an action against the friend for repayment of the loan? Answers: No, because the lender released her mortgage interest in the home. No, because the third party is the current owner of record of the home. Yes, because the friend executed a promissory note, which was payable to the lender. Yes, because the third party did not assume personal liability for loan.

Answer choice A is correct. Despite the sale of the home by the friend to a third party, the friend remained liable on the note. Because the sale documents did not make mention of the third party's liability with respect to the note, the third party did not assume liability for the note, but instead merely took the home subject to the mortgage. However, the friend is released from personal liability to the lender because the lender's release of the mortgage eliminates the friend's ability to be subrogated to the lender's mortgage interest if the lender forces the friend to pay the loan obligation. Since the lender's release of the mortgage removes this protection, the friend's obligation is discharged as a result. Answer choice B is incorrect because the third party's current ownership of the home does not absolve the friend of liability with respect to the note. Answer choice C is incorrect because, although the friend is liable to the lender for payment of the note, that liability is discharged due to the release by the lender of the mortgage on the home. Answer choice D is incorrect. Because the sale documents did not make mention of the third party's liability with respect to the note, the third party did not assume liability for the note, but instead merely took the home subject to the mortgage, Nevertheless, the friend's liability is discharged due to the release by the lender of the mortgage on the home.

Question 3154 A man purchased undeveloped land with a bank loan secured by a mortgage on the property. The deed and mortgage were promptly recorded. A year later, the man decided to sell the property to a wealthy widower. The widower purchased the property, recorded his interest, and assumed the mortgage. Several years later, the widower gave the property to his daughter. The widower did not tell his daughter about the mortgage but instead continued to make the mortgage payments. The deed, which contained no mention of the mortgage, was promptly recorded by the daughter. When the widower died, he devised all of his real property to his daughter. He left the remainder of his estate to his son. Following the widower's death, no one made payments on the loan, causing it to fall into default. May the bank foreclose on the property? Answers: Yes, because the bank recorded its mortgage. Yes, because the daughter received the property as a gift. No, because the daughter recorded her deed that made no mention of the mortgage. No, because of the exoneration of liens doctrine.

Answer choice A is correct. Every jurisdiction has enacted a recording statute. However, if the recording act does not govern, the common law first in time, first in right rule generally applies to determine priorities between competing interests in land. All of the recording acts require that the later grantee acquired the property for value in order to be protected by the act. Here, the daughter received the land as a gift, so she cannot claim protection of the recording act, and the common law first in time rule gives priority to bank. This analysis seems to implicate answer choice B, however that is not the end of the inquiry. Under the shelter rule, a grantor who is protected by the act can pass that protection on to a grantee who would otherwise by unprotected. Here, the daughter is not protected by the shelter rule because the widower is, himself, not protected by the recording act. Because the bank properly recorded its mortgage prior to the widower's purchase of the property, the widower was (i) on notice of the mortgage, and/or (ii) failed to record first. Thus, under any recording act, the widower is not protected because the bank properly recorded its interest. It is for this reason that the daughter is also not protected under the shelter rule. Answer choice B is incorrect because the manner in which the daughter acquired the property is irrelevant. The bank would have prevailed even had the daughter purchased the land from the widower because the bank recorded its mortgage prior to the man's transfer of the land to the widower, providing notice and/or recording first under any type of recording act. Answer choice C is incorrect because although the daughter had no personal knowledge of the mortgage nor did the deed reference the mortgage, she took the property subject to the recorded mortgage. Answer choice D is incorrect because the exoneration of liens doctrine does not apply to the mortgage on the land. The land was transferred to the daughter prior the widower's death; it did not pass to the daughter through the widower's will.

Question 5210 A business owner borrowed money from a financial institution in order to expand his business. The business owner executed a nonnegotiable promissory note to evidence his personal liability to repay the financial institution. In addition, the business owner granted the financial institution a mortgage on his condominium that was evidenced by a written document as security for the loan. By a separate written document, the financial institution assigned its interest in the note to a third party. This document made no mention of the mortgage. What is the effect of this transaction on ownership of the mortgage? Answers: The third party owns the mortgage, because the mortgage follows the note. The financial institution owns the mortgage, because there was no mention of the mortgage in the document assigning the note to the third party. The financial institution owns the mortgage, because a transfer of the note without the mortgage is void. The mortgage is discharged by the independent transfer of the note.

Answer choice A is correct. If the promissory note is transferred without the mortgage, the mortgage is treated as having been transferred as well under the principle that the mortgage follows the note. Answer choice B is incorrect because, although the written assignment of the note made no mention of the mortgage, because the mortgage serves as security for the note, ownership of the mortgage is deemed to follow the note. Answer choice C is incorrect because, while in some states the transfer of a mortgage without the transfer of the note is void, the transfer of the note is treated as also transferring the mortgage. Answer choice D is incorrect because the transfer of the note by itself is generally deemed to result in the transfer of the mortgage as well.

Question 4267 To pay for a home in the mountains, a retiree executed a mortgage with a seller. Due to a clerical error, the mortgage was never recorded. Years later, the retiree executed a second mortgage on the home with a bank to make improvements, and that mortgage was immediately recorded; as part of the mortgage approval process, the retiree had to disclose his first mortgage on the property. Shortly thereafter, the seller discovered that his mortgage had not been recorded, and recorded it. A year later, the retiree executed a third mortgage on the home after he realized the full effect his retirement was having on his finances. After the retiree went nearly a year without making mortgage payments, a foreclosure action was initiated. In a race-notice jurisdiction, which of the following is a correct statement about the first mortgage? Answers: It has priority over the second and third mortgages on the home. It has priority over the third mortgage on the home only. It is the most junior mortgage. The recording was invalid because the retiree had already executed a second mortgage on the land.

Answer choice A is correct. Priority of the mortgages will be determined by the recording statute. In this case, the jurisdiction has a race-notice recording statute. A race-notice statute requires a subsequent purchaser to take the interest without notice of a prior conflicting interest and be the first to record. Here, although the bank recorded its mortgage before the seller recorded the first mortgage, the first mortgage has priority because the bank had notice of the first mortgage when it entered into the second mortgage on the same property. Once the seller recorded the first mortgage, the seller was protected under the recording act and the first mortgage had priority over all subsequent mortgages, including the third mortgage on the home. Answer choice B is incorrect because the first mortgage also has priority over the second mortgage on the home. Answer choice C is incorrect because it is actually the most senior mortgage. Answer choice D is incorrect because no such restriction exists.

Question 1895 A widow executes a will in which she leaves her house to her son and the remainder of her estate to her daughter. The house is subject to a purchase money mortgage, the unpaid portion of which is nearly equal to the value of the residuary estate. The son demands that the personal representative of the estate use the residuary estate to pay off the mortgage. The will contains a general provision for the payment of all the testator's debts, but not a specific provision authorizing the payment of the outstanding balance of the mortgage. The jurisdiction follows the common law. Should the personal representative accede to the son's demand? Answers: Yes, because the son has a right to the exoneration of the mortgage. Yes, because the will contains a general provision for the payment of the testator's debts. No, because the doctrine of satisfaction does not apply to a specific devise. No, because the mortgage is a purchase-money mortgage.

Answer choice A is correct. The devisee of real property is entitled under the common-law doctrine of exoneration of liens to have any outstanding balance of a mortgage or other encumbrance on the property to be paid from the remaining assets of the testator's estate. Note that today, most jurisdictions have abolished this doctrine and property passes subject to any encumbrance. Answer choice B is incorrect because a general provision that requires the payment of a testator's debts does not require the personal representative of the estate to pay off any mortgages or other encumbrances on real property of the testator. Answer choice C is incorrect because the doctrine of satisfaction refers to the receipt of a gift from the testator prior to death that satisfies a devise made by the testator in the testator's will. Answer choice D is incorrect because the doctrine of exoneration of liens applies to all encumbrances, not just purchase-money mortgages.

Question 6440 A man owned a building. He executed a deed conveying the building to a local church "for the purpose of using the building to further religious education." Six years later, the man died, leaving his niece as his sole heir. The man's duly probated will left his entire estate to a friend. Eighteen months later, the local church, having never made use of the building, conveyed all of its interest in the building to an investor for valuable consideration. The investor has filed an action to quiet title against the friend and niece. The investor has also joined a state official who argues that a valid charitable trust was created, and that the attorney general of the state should be permitted to enforce the charitable trust. In whom should the court find proper title is vested? Answers: The investor The friend The state official You Selected: The niece

Answer choice A is correct. The man conveyed the building to the church in fee simple because the language "for the purpose of using the building to further religious education" is precatory language that only limits the property's purpose, rather than conditional language (e.g., "so long as") that is necessary to create a defeasible fee. The church then conveyed its interest in the building to the investor, who now possesses title to the building outright. Answer choice B is incorrect. The man's will did not devise the building to his friend because he no longer owned the building at his death. Additionally, because the man had conveyed the building in fee simple, the man did not retain a future interest that was devised to his friend. Answer choice C is incorrect. A charitable trust is one with a stated charitable purpose made to benefit the community at large or a particular segment of the community. In a charitable trust, the beneficiaries must be reasonably numerous and unidentifiable. On the other hand, if the language of a conveyance limits only the purpose for which the property is to be used, as does the conveyance by the man to the church in this case, it is treated as transferring a fee simple absolute interest in the property. Answer choice D is incorrect. Even if the man retained a reversionary interest in the building, his niece would not be entitled to it because the man left a valid will devising his entire estate to his friend.

Question 4213 A landowner gave her property's mineral rights to her son. After the transfer but before the son began to mine the minerals, the landowner sold the property to a corporation that built a commercial warehouse on the property. As a consequence of the son's subsequent mining activities, which were conducted with reasonable care and in compliance with all laws and regulations, the land subsided and the warehouse was damaged. Can the corporation recover for the damage to its warehouse? Answers: No, because the son exercised reasonable care in the conduct of the mining activities. No, because the warehouse was used for commercial rather than residential purposes. Yes, because the corporation acquired the property by purchase rather than gift. Yes, because the corporation built the structure before the son began to mine the minerals.

Answer choice A is correct. The owner of mineral rights is not liable for damage done to structures built on the surface of the land after the mineral rights were transferred unless the owner fails to exercise reasonable care in removing the minerals. Because the facts indicate the son did exercise reasonable care in conducting the mining operation, he is not liable for the damage to the warehouse. Answer choice B is incorrect because the right to subjacent support with regard to structures on the land does not turn on whether the structure was for residential or commercial use, but rather on when it was constructed. Answer choice C is incorrect because the manner of acquisition of the land does not affect the right of the owner of the land to subjacent support with regard to underground mining activities. Answer choice D is incorrect because the right to subjacent support with regard to structures on the land does not turn on when the mining began. If the structure was constructed before the rights were acquired, the owner of the mining rights is strictly liable for the failure to provide subjacent support. If the structure was constructed after the rights were acquired, as is the case here, the owner of mining rights is liable for the failure to provide subjacent support only if the owner failure to exercise reasonable care in removing the minerals. Here, the construction began after the rights were acquired; it is irrelevant that the son had yet to begin extracting the minerals.

Question 6429 Fifteen years ago, a farmer purchased a large undeveloped tract of land. The land was located next to a state-run automobile salvage yard that had already been located there for more than a decade. Neither the salvage yard nor the farmer ever surveyed their properties to clearly identify or mark the boundary between their tracts. Over the last fifteen years, the salvage yard has started to spread onto the adjacent edge of the farmer's land, regularly occupying a ten-yard-wide strip of the property. Neither the farmer nor the salvage yard ever knew that the scrap heaps were regularly encroaching on the farmer's property. When the farmer died last year, his son inherited the land and planned to develop the property as a farm. When he surveyed the property, the trespass was discovered. The time for acquiring title by adverse possession in the jurisdiction is ten years. The son has demanded that the salvage yard cease placing scrap heaps on the son's land. When the salvage yard refused his demand, the son brought an action to enjoin any such use in the future. Of the following, what is the salvage yard's best argument to defeat this action? Answers: The salvage yard has acquired title to the ten-yard-wide strip of the son's property through adverse possession. The state can properly exercise the power of eminent domain to take the ten-yard-wide strip of property. The farmer impliedly consented to the salvage yard's use of the land by failing to object to the scrap heaps on his property. The balance of the equities weighs in favor of the state-run salvage yard.

Answer choice A is correct. Under the doctrine of adverse possession, ownership of real property is transferred to a person who exercises exclusive physical possession of that property for a specific amount of time. This is the salvage yard's strongest defense and is the only argument provided that could result in a judgment for the salvage yard on these facts. Answer choice B is incorrect. Even if this unintentional trespass by a state-run salvage yard reflected a legitimate state interest sufficient to justify a government taking of this strip of land, the son would be able to recover damages for the taking. Therefore, this is not as strong a defense as adverse possession, which would not result in monetary damages for the son. Answer choice C is incorrect because this argument would not support a holding for the salvage yard. Even if the farmer's lack of knowledge of the trespass had indicated his consent, the son now owns the property and will be able to revoke the implied consent. Answer choice D is incorrect. Although a court in determining whether to issue an injunction must balance the equities, it is at best arguable that the use of this strip of land to store scrap heaps should take precedent over the claims of the farmer's son to the land. Moreover, if the use of the strip for a public purpose is required, then the state is required to do so through the mechanism of eminent domain.

Question 6620 An accountant had only two children—a clerk and a doctor. The accountant made an inter vivos conveyance of land to a banker, "for the life of the banker, and then to my heirs; but if none of my heirs survive the banker, then to my lawyer." Two months later, the accountant died, leaving the clerk and the doctor as his only heirs. Recently, the doctor died. The banker is still living. The jurisdiction does not apply the Rule in Shelley's Case or the doctrine of worthier title. Of the following, which best describes the clerk's current property interest in the land? Answers: The clerk has a vested remainder subject to complete divestment. The clerk has a vested remainder subject to open. The clerk has a contingent remainder. The clerk has no interest in the land.

Answer choice A is correct. When the accountant transferred ownership of the land, the banker took a life estate, and, since the doctrine of worthier title does not apply, the accountant's heirs had a contingent remainder. This remainder was contingent because, since the accountant was still alive, his heirs were not then ascertainable. The clerk had a mere expectancy in the land because he was not an heir of the accountant at that time. When the accountant died, the clerk's interest, as the accountant's heir, became a vested remainder. This remainder is subject to complete divestment by the lawyer, who has an executory interest, if the clerk dies before the banker, who is still living. Answer choice B is incorrect. Although the clerk did acquire a vested remainder interest in the land upon the death of the accountant as one of the two heirs of the accountant, this remainder interest was not subject to open. The only individuals who could become heirs of the accountant were determined at the time of the accountant's death. Answer choice C is incorrect. A remainder is contingent if it is created in a grantee that is unascertainable, or if it is subject to an express condition precedent to a grantee's taking. Although the remainder interest in the accountant's heirs was contingent at the time it was created because the accountant was alive and his heirs were not then ascertainable, the clerk himself did not have an interest in the land at the time. He had a mere expectancy as a potential heir of the accountant. Upon the accountant's death, the clerk, as an heir of the accountant, held a vested remainder in the land subject to complete divestment if he does not survive the banker. Answer choice D is also incorrect. In states that apply the doctrine of worthier title, the doctrine cuts off remainders in the heirs of the transferor. In such states, the conveyance of a life estate followed by a remainder to the "grantor's heirs" is treated as creating a reversion in fee simple in the grantor, instead of a remainder interest in the grantor's heirs. However, this jurisdiction does not adhere to this doctrine.

Question 4385 An attorney was a sole practitioner specializing in family law. Her niece was a recent law school graduate, and her nephew was an attorney. The attorney decided to retire, and conveyed the historic building that housed her law practice "to my niece, but if she fails to pass the bar exam within a year of her law school graduation, to my nephew." Which of the following is an accurate description of the property interests created? Answers: The niece has a fee simple subject to condition subsequent, the nephew has a right of re-entry, and the attorney has no interest. The niece has a fee simple subject to an executory interest, the nephew has an executory interest, and the attorney has no interest. The niece has a fee simple determinable, and the attorney and the nephew each have a possibility of reverter. The niece has a fee simple subject to condition subsequent, the attorney has a right of re-entry, and the nephew has an executory interest.

Answer choice B is correct. A fee simple subject to an executory interest (sometimes referred to as a "fee simple subject to an executory limitation") is a present fee simple estate that is limited in duration by specific conditional language (e.g., "provided that," "on condition that," "but if") such that, upon the occurrence of the specified condition, title will automatically pass to a third party (i.e., someone other than the grantor or the holder of the present fee). The future interest held by the third party is an executory interest. Unlike the corresponding future interest in the grantor (right of entry), the executory interest automatically comes into existence; there is no need for the third party to take any action (e.g., an eviction action). In this case, the niece had a fee simple subject to an executory interest due to the conditional language in the grant. The nephew had an executory interest that would become a possessory interest if the niece did not pass the bar exam. Answer choices A and D are incorrect because the title would pass to a third party, rather than the grantor. A fee simple subject to a condition subsequent is a present fee simple that is limited in duration by specific conditional language, such as "provided that." With a fee simple subject to condition subsequent, the grantor may terminate the estate upon occurrence of the stated condition. Here, the grantor did not retain the right to terminate the estate, but rather gave the nephew an executory interest. Answer choice C is incorrect. A fee simple determinable is a present fee simple estate that is limited by specific durational language (e.g., "so long as," "while," "during," "until"), such that it terminates automatically upon the happening of a stated condition, and full ownership of the property is returned to the grantor (or his successor in interest). In this case, the grant contained conditional language (i.e., "but if"), indicating a fee simple subject to a condition subsequent or subject to an executory interest. Because the attorney (the grantor) did not retain the right to terminate the estate, and instead gave the nephew an executory interest, the grant was a fee simple subject to an executory interest. Accordingly, neither the attorney nor the nephew had a possibility of reverter.

Question 4212 A landowner sold the right to access and remove oil from below the surface of the property to a company. As a consequence of the removal of the oil, the ground subsided, although the company exercised reasonable care and complied with all laws and regulations. The landowner's residence, which had been on the property long before the landowner purchased the land, was damaged due to the subsidence. Can the landowner recover from the company for the damage to his residence? Answers: Yes, because the property damaged was a personal residence. Yes, because the residence predated the company's oil rights. No, because the company exercised reasonable care in removing the oil. No, because the company complied with all laws and regulations.

Answer choice B is correct. A landowner enjoys the right to subjacent support when the landowner transfers the rights to access and remove oil or minerals from the property. The transferee of such rights is strictly liable for any damage that occurs to existing structures on the land as a consequence of subsidence caused by the removal of the oil or minerals. Answer choice A is incorrect because the right to subjacent support with regard to structures on the land does not turn on the type of structure on the land, but on the time of construction of the structure in relationship to the time of the transfer of the mineral rights. Answer choice C is incorrect because the exercise of care by the company does not protect the company from strict liability. Answer choice D is incorrect because, although compliance with all laws and regulations may demonstrate the reasonableness of the company's actions in removing the oil, it does not protect the company from strict liability with regard to existing structures.

Question 5204 The owner of a retail store sold the store to two of her employees, the manager and the bookkeeper. The employees took ownership of the store as tenants in common with equal ownership interests. At the time of the sale, there was an existing mortgage on the store that the owner had granted to a bank in exchange for a loan. The manager and the bookkeeper did not assume the obligation to repay the loan. Six months later, the former owner became insolvent and the loan went into default. Responding to the bank's threat to foreclose on the mortgage, the manager paid off the loan. Can the manager enforce the mortgage against the bookkeeper's interest in the store? Answers: Yes, because the manager could not recover from the former owner due to the former owner's insolvency. Yes, but only to the extent of one-half of the payment made by the manager. No, because the bookkeeper did not assume the obligation to repay the mortgage loan. No, because the mortgage was extinguished by the manager's payment of loan for which the mortgage served as security.

Answer choice B is correct. A person who pays off a loan that is secured by a mortgage in order to protect her own interests acquires the rights of the original mortgagee-lender and may therefore enforce the mortgage. This concept is called "subrogation." As a tenant in common with an equal ownership interest in the store with the bookkeeper, the manager is entitled to contribution from the bookkeeper for one half of the payment made by the manager because the payment protected the manager's interest in the store as well as the bookkeeper's. Answer choice A is incorrect. The manager, having paid off the loan to protect his own interest in the store, is entitled to seek recovery from the former owner of the store as the person primarily liable for payment of the loan. While the former owner's insolvency makes this avenue unlikely to be worth pursuing, there is no requirement that the manager pursue this option first or establish its futility before seeking to enforce the mortgage itself. The mortgagee can enforce a mortgage without first seeking the personal liability of the mortgagor. Answer choice C is incorrect because, although the bookkeeper did not assume the obligation to repay the mortgage loan, the store, including the bookkeeper's half interest in it, was subject to the mortgage. Permitting the bookkeeper to retain that interest without compensating the manager to the extent that the manager's payment of the loan benefited the bookkeeper would result in the unjust enrichment of the bookkeeper. Answer choice D is incorrect because, although generally the payment in full of a loan for which a mortgage serves as security results in the extinguishment of the mortgage, in this case the manager's payment of the loan resulted in the manager becoming subrogated to the bank's rights to enforce the mortgage.

Question 4393 In 1959, a man owned a tract of land that he divided into two parcels, each of which was adjacent to a stone retaining wall. The man sold both parcels. In the deeds that created the parcels, the purchasers mutually agreed that: "the owners, their heirs, and assigns agree to maintain a retaining wall made of stone between the properties. They further agree that they will share equally any expenses associated with the retaining wall." The deed was properly recorded. Subsequently, each of the parcels was sold several times, and none of the owners properly maintained the retaining wall. In 2009, the owners of the parcels made the joint decision to dismantle the wall. In 2012, one of the parcels was sold, and the new owner decided to rebuild the retaining wall. The new owner asked the other owner to pay half of the expenses to rebuild the wall, and the other owner refused. The new owner paid to erect the retaining wall, and then brought suit against the other owner to collect half the expenses. Can the new owner successfully bring suit against the other owner for half of the expenses? Answers: No, because the covenant was effectively terminated by a change in circumstances. No, because the previous owners decided to dismantle the retaining wall. Yes, because the deed created an equitable servitude. Yes, because the covenant ran with the land.

Answer choice B is correct. In order for a covenant to be enforceable it must be in writing, with the intent that the duties and rights run with the land, must touch and concern the land, subsequent purchasers must have notice of the covenant (either actual, constructive, or inquiry notice), and the parties must have horizontal privity (the estate and the covenant are contained in the same instrument) and vertical privity (an unbroken chain of ownership from the original parties). A covenant may be terminated in the same ways as an easement, including by abandonment. A covenant can be terminated by abandonment if the parties to the covenant act in an affirmative way that shows a clear intent to relinquish the covenant right; mere nonuse, however, or statements of intent without affirmative conduct, will not constitute abandonment. Here, the parties satisfied the criteria for creating a covenant appurtenant in 1959. The covenant was then abandoned by subsequent owners who dismantled the retaining wall that the covenant was created to maintain. Answer choice A is incorrect because there was not a drastic change in circumstances that would justify not enforcing the covenant. If a restriction (i.e., real covenant or equitable servitude) on a property no longer makes sense to enforce due to drastic changes in the surrounding area since the restriction was first contemplated, it will not be enforced. The facts in this case do not indicate that there was any such drastic change. Answer choice C is incorrect because only equitable remedies such as injunctions are available to the holder of an equitable servitude. In this case, the new owner is seeking money damages, which are available only for breach of a covenant appurtenant. Answer choice D is incorrect because, although the covenant ran with the land, it was subsequently terminated by abandonment.

Question 5865 Upon her death, the owner of six acres of undeveloped land devised her property to her brother and sister as "joint tenants with the right of survivorship" with a 1/3 interest in the land allocated to the owner's brother and a 2/3 interest in the land allocated to the owner's sister. Neither the brother nor the sister transferred their interest in the land during their lifetime. The brother, upon his death, willed his interest in the land to a friend. The sister later died intestate. Her sole heir inherited all of her property. Who owns the undeveloped land? Answers: The sister's sole heir owns the six acres of undeveloped land outright. The sister's sole heir owns a 2/3 interest and the brother's friend owns a 1/3 interest in the land as tenants in common. The sister's sole heir owns four acres of the land, and the brother's friend owns two acres of the land, each in fee simple absolute. The sister's sole heir owns a 2/3 interest and the brother's friend owns a 1/3 interest in the land as joint tenants with the right of survivorship.

Answer choice B is correct. Although the owner used the term "joint tenants with the right of survivorship" in devising the undeveloped land to her brother and sister, each held their interest in the property as tenants in common with one another. They did not own equal shares of the land because the sister was devised a 2/3 interest, while the brother was devised a 1/3 interest. Thus, they did not satisfy the unity of interest requirement to holding the property as joint tenants with the right of survivorship. However, because the owner did indicate by use of the term "joint tenants with the right of survivorship" that she wanted her brother and sister to own the land jointly, they took their respective interests as tenants in common. A tenancy in common is not subject to the right of survivorship. Consequently, upon the brother's death, his 1/3 interest in the land passed pursuant to the terms of his will to his friend. Upon the sister's death, her 2/3 interest in the land passed in accord with the intestacy laws to her sole heir. Answer choice A is incorrect because as discussed above, the owner's brother and sister held their interests in the undeveloped land as tenants in common, thus there is no right of survivorship. Had the undeveloped land been held as a joint tenancy with the right of survivorship, the sister's sole heir would own the property outright. Answer choice C is incorrect. The brother and sister took their respective interests as tenants in common because the owner indicated by use of the term "joint tenants with the right of survivorship" that she wanted her brother and sister to own the land jointly. As tenants in common, each person had an undivided share of the entire six acres, a common characteristic of concurrently held property. Thus, neither person owned a specific acreage. Answer choice D is incorrect. Although the owner used the term "joint tenants the right of survivorship" in devising the undeveloped land to her brother and sister, each sibling held their interest in the property as a tenant in common with one another. Since they did not own equal shares of the land, they did not satisfy the unity of interest requirement to holding the property as joint tenants with the right of survivorship.

Question 3136 A widower owned a residence in fee simple absolute. His only child, a daughter, also had only one child, a son, who had a child, Ann. The widower executed a will in which the residence was devised to his daughter for her life, and then remainder to his grandson's children. The widower left the rest of his estate to a charity. After the widower's death, his grandson had a second child, Bill. Subsequently, the widower's daughter died. A year later, the grandson had a third child, Claire. Recently, the widower's grandson died. All three of the grandson's children have survived him. The applicable jurisdiction continues to follow the common-law Rule Against Perpetuities as well as the Rule of Convenience. Who now owns the residence? Answers: Ann Ann and Bill Ann, Bill, and Claire The charity named in the will.

Answer choice B is correct. At the time of the widower's (W's) death, his daughter (D) had a life estate interest in the residence, Ann (A) had a vested remainder subject to open, and the grandson's (G's) unborn children (Bill and Claire, B and C) had contingent remainders in the residence. Just prior to the death of the widower's daughter, G's living children, A and B, each had a vested remainder subject to open. Because the class of the G's children had members at the time that it became possessory (upon the death of D), the class closed at that time because the jurisdiction applies the Rule of Convenience. Consequently, A and B are part of the class, but C, being born after the class closed, is not entitled to share ownership of the residence with her siblings. Answer choice A is incorrect because, although Ann's remainder interest vested upon the widower's death, her interest was subject to being shared with subsequent siblings. Even though B was born after W's death, the class did not close until D's death (when the interest became possessory). Thus, B also owns the residence. Answer choice C is incorrect because C was born after the class closed. Answer choice D is incorrect because it assumes the devise violated the Rule Against Perpetuities (RAP). As a remainder interest, the devise to the G's children was subject to the RAP, and the residence would have passed to the charity, as the owner of the rest of the widower's estate, if the RAP voided the devise. However, RAP was not violated, because, while the widower's grandson (G) could have had another child more than 21 years after the death of D (who was a measuring life), G, a person who was alive at time that the remainder interest was created and on whom his children's existence obviously depended, is also a measuring life. Since the children of the grandson were fixed upon his death, the grandson was also a validating life with respect to the remainder interest in his children. While distribution of the remainder interest among A, B, and C would not violate RAP, the Rule of Convenience required, as discussed with regard to answer choice B, that the class closed upon the death of D before C was born. Thus, A and B are entitled to the residence.

Question 5209 A homebuyer financed the purchase of her home with a loan from a bank, granting the bank a mortgage in the home to secure her loan obligation. The bank promptly recorded its mortgage interest. A few years later, the homebuyer borrowed money from a finance company to pay for a child's tuition expenses. The finance company took a mortgage in the home, which it promptly recorded. Subsequently, the homebuyer, while maintaining payments to the finance company, defaulted on her bank loan. The bank initiated foreclosure proceedings against the homebuyer. The finance company did not receive notice of these proceedings. The home was sold at a judicial foreclosure sale to an investor who intended to rent out the home. Has the finance company's mortgage in the home been eliminated? Answers: No, because the finance company's loan was not in default at the time of the foreclosure sale. No, because the finance company did not receive notice of the foreclosure sale. Yes, because the finance company's mortgage was a junior mortgage. Yes, because the home has been sold at a judicial foreclosure sale.

Answer choice B is correct. Because the finance company took a mortgage in the home after the bank took and recorded its mortgage in the home, the finance company's mortgage is a junior mortgage over which the bank's mortgage has priority. Generally, the foreclosure of a senior mortgage by a sale eliminates any junior mortgages. However, when a junior mortgagee is not given notice of the foreclosure proceedings, the junior mortgage is not eliminated. Answer choice A is incorrect because the foreclosure on a senior mortgage eliminates a junior mortgage, regardless of whether the junior mortgage is in default at that time. Answer choice C is incorrect because, although the finance company's mortgage was a junior mortgage because it was taken after the bank took and recorded its mortgage, it was not eliminated by the foreclosure sale because the finance company did not receive notice of the foreclosure proceedings. Answer choice D is incorrect. While the home has been sold at a foreclosure sale, the junior mortgage was not eliminated because the junior mortgagee did not receive notice of the sale.

Question 5202 A homebuyer borrowed money to finance her purchase of a residence. The homebuyer executed a promissory note, payable to the lender, in the amount of the loan plus interest and gave the lender a mortgage on the residence as security for repayment of the loan. Subsequently, the homebuyer sold the residence to a friend. As part of the sales agreement between the homebuyer and her friend, the friend assumed liability for payment of the note. Later, when the value of the home exceeded the outstanding obligation on the note, the lender released its mortgage interest in the home. Shortly thereafter, the loan fell into default. Can the lender successfully maintain an action against the original homebuyer for repayment of the loan? Answers: No, because the friend assumed personal liability for payment of the note. No, because the lender released its mortgage interest in the home. Yes, because the homebuyer executed a promissory note, which was payable to the lender. Yes, because, upon repayment of the loan, the original homebuyer is subrogated to the lender's rights against the friend with respect to the note.

Answer choice B is correct. Despite the homebuyer's sale of the residence to a friend and the friend's assumption of liability on the note, the homebuyer remained liable on the note. By releasing the mortgage, the lender has eliminated the homebuyer's recourse to the mortgage to recover, at least to the extent of the value of the residence, the amount that the homebuyer is obligated to pay the lender pursuant to the note. Consequently, the homebuyer's obligation is discharged as a result of the lender's release of the mortgage. This is true even though the homebuyer also has recourse to seek the personal liability of her friend because of the assumption agreement between them. Answer choice A is incorrect. Even though the friend did assume personal liability for payment of the note, this assumption agreement, through which the friend became primarily liable for payment of the note and the homebuyer secondarily liable as a surety, was only between the homebuyer and her friend. Consequently, the lender can maintain an action to enforce the note against the homebuyer without first seeking recovery from the friend. However, such an action will not be successful because the lender, by releasing the mortgage, has eliminated the homebuyer's recourse to the mortgage. Answer choice C is incorrect because, even though the lender can seek to enforce the note against the homebuyer, such an action will not be successful as the lender has destroyed the homebuyer's recourse to the mortgage. Answer choice D is incorrect because, even though the homebuyer would be subrogated to the lender's rights against the friend with respect to the note if the homebuyer repaid the loan, the homebuyer's personal obligation to the lender was discharged when the lender released the mortgage.

Question 6222 A daycare business was operated by its owner out of a rented building for eight years. The building that housed the daycare operation was located next to a fenced-in playground. Under the daycare owner's direction, the children at the daycare facility used the playground five days a week. The daycare owner did not own the parcel upon which the playground was located nor did she know who did. Had she asked, the owner of the land would have given permission for the children to use the playground. To keep vagrants and other people out, the daycare owner padlocked the playground when not used by the daycare children. When the daycare owner suddenly fell into a coma, a friend continued to operate the daycare facility as the owner had for the next four years. Recently, the daycare owner died. Shortly thereafter, the owner of the playground parcel returned, introduced himself to the daycare owner's friend, and revealed to her his plans to build a high-rise condominium on the playground parcel. The friend has filed a suit seeking a declaration that she is the owner of the parcel. The statutory period to obtain title by adverse possession in the jurisdiction is 10 years. Of the following, which is the best defense that the owner of the playground parcel can make against the lawsuit? Answers: The landowner would have granted the daycare facility permission to use the playground had it been sought. The friend has not possessed the playground for the statutory period. The daycare owner and her friend only rented the property on which the daycare business operated. The playground lot was used by the daycare business for only five days each week.

Answer choice B is correct. To obtain title by adverse possession, the possession must be open, notorious, continuous, hostile, exclusive, and for the requisite statutory period. Here, the use of the playground was open, notorious and continuous, the owner of the playground parcel did not give his permission, and other people were prevented from accessing the fenced-in playground because of the padlock. However, because the friend has possessed the playground parcel for only four years, her possession of the playground parcel does not satisfy the statutory period of ten years. Although the friend can argue that the use of the playground parcel by the daycare operator for eight years should be tacked onto her own use for four years, it is unclear whether there is sufficient privity of interest to permit such tacking. The two shared the same economic motivation for using the playground lot (i.e., to advance the interest of the daycare operation). However, it is unclear whether the friend was acting in a non-hostile manner as a surrogate for the original daycare operator or in a hostile manner running the daycare operation for her own benefit. Accordingly, answer choice B is the best defense for the owner of the playground parcel (even if, ultimately, it might not be successful). Answer choice A is incorrect. Because neither the original owner-operator nor her friend sought permission to use the playground lot from its owner, their use of the lot was hostile. It is irrelevant that the owner of the playground parcel would have given permission had he been asked. Answer choice C is incorrect. Although the establishment of a prescriptive easement appurtenant would require that the adverse user have ownership of the dominant estate, there is no requirement that, in order to qualify as an adverse possessor of real property, a person must own adjoining property. Answer choice D is incorrect because the original owner-operator and her friend used the playground lot as a true owner would. Although the parcel was used by children from the daycare for five days a week, the parcel was padlocked the remaining time in order to deny anyone else access to the parcel.

Question 3157 An investor purchased undeveloped land with the aid of a loan from a bank. The loan, which was evidenced by a note, was secured by a mortgage on the property. The note contained a "due on sale" clause. Subsequently, the investor sold the land to a developer. The bank agreed to waive the "due on sale" clause if the developer assumed the mortgage, which the developer did; the waiver was silent as to the note. The following year, the developer failed to make timely payments on the note, resulting in a default. The developer filed for bankruptcy, and his personal liability for the note was completely discharged. Would the bank be successful in an action against the investor based on the default of note? Answers: Yes, because the developer is no longer liable on the note. Yes, because the investor is liable on the note. No, because the bank waived the "due on sale" clause. No, because the developer assumed the mortgage.

Answer choice B is correct. Following the transfer of a mortgage, the investor, as maker of the note, remains liable on the note absent a release by the bank or other act that would discharge the investor's liability. Answer choice A is incorrect because the investor's liability to the bank for the default does not depend on the developer's liability. A discharge of the developer does not discharge the investor. Answer choice C is incorrect because although the bank waived the "due on sale" clause, which would have triggered the investor's liability for the full amount of the loan upon the sale of the land, the bank did not release the investor from liability on the note. Answer choice D is incorrect because assumption of a mortgage by a transferee of the mortgaged property does not eliminate the transferor's liability, but instead makes the transferee also liable to the mortgagee for payment of the note. The elimination of the transferee's personal liability through bankruptcy does not absolve the transferor of liability with respect to the note.

Question 5957 A homeowner bought a home with the proceeds of a loan from a thrift institution. The loan was secured by a mortgage on the home. Under the terms of the loan, the full amount of the outstanding loan obligation was to become due and payable if the home was sold or otherwise transferred without the prior permission of the thrift institution. The thrift institution recorded its mortgage. Subsequently, the homeowner established a living trust and transferred ownership of her home to the living trust. The homeowner recorded this ownership transfer. Upon learning of the transfer, the thrift institution demanded that the homeowner pay the outstanding amount due on the loan immediately. When the homeowner refused, the thrift institution brought a foreclosure action to collect the full amount of the outstanding loan obligation. Is the thrift institution likely to succeed? Answers: No, because the transfer of ownership of the home to the living trust was recorded. No, because ownership of the home was transferred to the homeowner's living trust. Yes, because the thrift institution recorded its mortgage. Yes, because the thrift institution's mortgage is a purchase-money mortgage.

Answer choice B is correct. Generally, an acceleration clause (due-on-sale clause) in a mortgage loan document is enforceable. However, federal law provides a residential real property exemption, exempting certain transfers of residential real property from the requirement that states give effect to an acceleration clause. Among the exempt transfers is a transfer by the mortgagor-borrower to her living trust. Consequently, it is unlikely that the court will give effect to the acceleration clause in this situation and find the loan in default due to the transfer of ownership from the homeowner to her living trust. Answer choice A is incorrect because, while recording a transfer in ownership can give the transferee priority over a prior unrecorded transfer, the thrift institution recorded its mortgage. Moreover, the transferee, the living trust, was not a bona fide purchaser of the property who could claim protection under a recording statute. Answer choice C is incorrect. Although the recording of a mortgage interest can protect the mortgagee from ownership claims by other subsequent takers of the property, the mortgagee here cannot foreclose because the mortgagor is not in default. As noted with regard to answer choice A, the transfer of the home from the homeowner to his living trust does not trigger the acceleration clause. Answer choice D is incorrect because, although the fact that a mortgage is a purchase-money mortgage can give the mortgagee priority over the holders of non-purchase-money mortgages in the same property, in this instance, this status has no bearing on the applicability or enforcement of the acceleration clause.

Question 5952 A restaurant owner purchased kitchen equipment from a supplier and gave the supplier an unsecured, nonnegotiable promissory note. Prior to the date on which payment of the note was required, the owner asked the supplier for an extension of time to pay the note. The supplier demanded in exchange that the restaurant owner execute a deed of trust with respect to the restaurant itself to serve as security for payment of the note. When the restaurant owner refused this demand, the supplier threatened to falsely report health code violations by the restaurant owner to the local health department. Acting under duress, the restaurant owner executed the deed of trust, which the supplier promptly recorded. Shortly afterwards, the owner sold the restaurant to the restaurant's chef. The chef assumed the obligation to pay the promissory note. The cash amount paid by the chef to the owner was less than the fair market value of the restaurant because the cash payment reflected the assumed obligation. The note is now in default and the trustee has instituted foreclosure proceedings on the deed of trust. Can the chef assert the owner's defense of duress against the trustee? Answers: No, because the supplier recorded the deed of trust prior to the sale of the restaurant to the chef. No, because the cash amount paid by the chef reflected the assumption of the owner's obligation on the note. Yes, because, by assuming the owner's obligation, the chef is entitled to assert the owner's defenses. Yes, because duress is a defense to the enforcement of a mortgage.

Answer choice B is correct. If the purchase price reflects the assumption of the mortgage, the purchaser of a mortgaged property who assumes the mortgage cannot assert the defenses of the mortgagor-seller against the enforcement of the mortgage. Otherwise, the purchaser would, by being permitted to avoid the assumed obligation, be unjustly enriched. (Note: A deed of trust functions as a mortgage in many states.) Answer choice A is incorrect because the question asks about the chef's ability to assert the owner's duress defense. The supplier's recording of the deed of trust serves only to protect the supplier from any claim by the chef, that the purchaser's rights have priority over the supplier's deed of trust. Under these facts, the supplier's recording of its deed of trust is not relevant. Answer choice C is incorrect because permitting the chef to assert the owner's defenses would result in a windfall to the chef, because the chef's promise to assume payment of the note was part of the price that the chef paid for the restaurant. Answer choice D is incorrect because, although duress is a valid defense to the enforcement of a deed of trust or a mortgage, the chef, as transferee, cannot in this case assert a defense that belongs to the transferor because it would unjustly enrich him.

Question 5869 A corporate officer purchased a house with the aid of a loan at a favorable interest rate from the corporation. The corporation required the officer to grant it a mortgage as security for the loan. The officer also signed a note promising to repay the loan over a period of ten years. The note contained a due-on-sale clause, which required, at the corporation's option, payment of the full outstanding amount of the loan if the officer sold the house without first obtaining the corporation's written consent. After making timely payments on the loan for three years, the officer sold the house to an unrelated buyer without obtaining the corporation's consent. The deed given by the officer acknowledged that the house was being transferred subject to the mortgage. After payment of the purchase price to the officer, the buyer promptly recorded the deed. Subsequently, neither the officer nor the buyer made any payments on the loan to the corporation. The corporation has sued the buyer for the outstanding amount of the loan. Of the following, which is the buyer's best defense to this action? Answers: The due-on-sale clause is void as an impermissible restraint on alienation. The buyer is not personally liable for repayment of the outstanding amount of the loan. The buyer is not in privity of estate with the corporation. The officer remains personally liable for repayment of the outstanding amount of the loan.

Answer choice B is correct. If the transferee-buyer takes title "subject to" an existing mortgage obligation, then the transferee-buyer is not personally liable upon default. In this case, the buyer purchased the house subject to the mortgage held by the corporation. As a result, the buyer did not assume personal responsibility for the loan. Therefore, the buyer is not personally liable to the corporation for repayment of the mortgage loan. Answer choice A is incorrect because, although a due-on-sale clause may constitute a restraint on the alienation of property, such a clause is specifically permitted by federal law, subject to several restrictions on residential real property, none of which are applicable in this situation. Answer choice C is incorrect. It is true that the buyer is not in privity of estate with the corporation. Privity of estate requires both parties to an agreement to share an interest in the property that is the subject of the agreement. Here, there is no agreement between the buyer and the corporation. However, the fact that the buyer is not in privity of estate with the corporation has no effect on the buyer's liability for the outstanding amount of the loan. Answer choice D is incorrect. Although the officer remains personally liable for repayment of the outstanding amount of the loan, this liability does not automatically shield the buyer from personal liability with regard to the loan. Had the buyer agreed to assume the mortgage obligation, the buyer, as well as the officer, would have been personally liable for repayment of the outstanding amount of the loan.

Question 5873 A man decided to sell his house after receiving a new job in a neighboring state. Before putting the house on the market, the man told his friend, who had always said how much she enjoyed the house, that he was selling the house and she could buy it at a lower price than he would seek from other potential buyers. Excited at the prospect of home ownership and the lower price, the friend immediately agreed to purchase the house and entered into a contract with the man for the sale of the house, without inquiring as to any issues with the house the man had experienced or knew about. Although he did not say anything to the friend, the man was not aware of any issues. Pursuant to the contract, the man delivered a general warranty deed to the friend at the closing. The friend then moved into the house and decided she would hire a contractor to perform some slight renovations to the upstairs bathroom. As soon as the contractor broke through the wall, he discovered black mold all throughout the interior of the bathroom and along the pipes. Upon further inspection black mold was found behind the walls, throughout the upstairs. The friend brought suit against the man for damages. Pursuant to statute, a seller has a duty of disclosure in all home-sale transactions in the jurisdiction. For whom is the court likely to rule? Answers: The man, because the friend did not ask about the condition of the house. The man, because he was not aware of the black mold. The friend, because the covenants contained in the general warranty deed were breached. The friend, because the black mold constituted a material physical defect in the house.

Answer choice B is correct. In a majority of jurisdictions, including the jurisdiction here, a seller of a residence has a duty to disclose. The seller must disclose all known material physical defects to the buyer. The defect must not be readily observable or known to the buyer. In this case, the man was not aware of the presence of black mold in his house. Therefore, he did not violate the duty of disclosure imposed on him by statute. Answer choice A is incorrect. The man had a duty to disclose all known material physical defects to the friend, even if the friend did not first ask about the condition of the house. However, the man was not aware of the black mold, and accordingly, he cannot be held liable for failure to disclose. Answer choice C is incorrect. The six covenants of title that are contained in every general warranty deed only protect the buyer from defects in title, not defects in the condition of the home. Answer choice D is incorrect. To be material, a defect must substantially affect the value of the residence, impact the health or safety of a resident, or affect the desirability of the residence to the buyer. Although the presence of black mold falls within the definition of a material physical defect, the man will not be held liable here because that defect was not known.

Question 7399 An owner conveyed one of his properties to his son for the son's life, remainder to his daughter. The son lived on the property without paying any rent, although the property could have been rented for $4,000 a month. The property was assessed annual property taxes of $10,000. The son did not pay the taxes on the property. Not wanting to have a lien on the property or otherwise have it foreclosed upon, the daughter paid the property taxes. The fair market value of the life estate was 10 percent of the fair market value of the property held in fee simple absolute. How much can the daughter recover from the son for the tax payments? Answers: $10,000, because life tenants are responsible for paying the full amount of taxes assessed on the property. $10,000, based on the reasonable rental value of the property. $1,000, the amount of taxes owed based on the fair market value of the life estate. Nothing, because the property taxes are the responsibility of the holder of the remainder interest.

Answer choice B is correct. Life tenants have the obligation to pay all ordinary taxes on the land and interest on the mortgage to the extent he receives a financial benefit from the property. When the life tenant occupies the property, his financial benefit is measured by the fair market rental value of the property. When the holder of a future interest pays the taxes because the life tenant fails to do so, the holder can bring an action against the life tenant personally to recover these payments from the life tenant, up to the value of the financial benefit the life tenant received from the property. Because the property in question is not producing income but has a reasonable rental value of $48,000 per year ($4,000 per month), the life tenant is liable to the holder of the remainder interest for the full amount of the taxes paid by the holder of the remainder interest, $10,000, as that amount does not exceed the reasonable rental value. Answer choice A is incorrect. Life tenants are not necessarily responsible for paying the full amount of taxes assessed on the property. Life tenants have the obligation to pay all ordinary taxes on the land to the extent he receives a financial benefit from the property. If the life tenant occupies the property, as in this case, the life tenant is responsible for taxes and mortgage interest to the extent of the reasonable rental value of the land. Answer choice C is incorrect because the amount of taxes owed by the son is not based on the fair market value of the son's life estate, but rather the reasonable rental value of the property. Answer choice D is incorrect because the life tenant owes the holder of a future interest a duty to pay the property taxes to the extent of the financial benefit he receives from the property.

Question 5205 On January 1, a contractor entered into an agreement with a bank to obtain financing for a construction project. The bank agreed to loan the contractor six million dollars, with one million dollars to be paid immediately and the remainder to be paid in million-dollar increments at two-month intervals. The contractor granted the bank a mortgage on land owned by the contractor in a different part of the state to serve as security for repayment of the amount loaned by the bank. The bank promptly recorded this mortgage. In May, after the bank had loaned the contractor a total of three million dollars, the contractor obtained a $500,000 loan from a private equity firm. This loan was also secured by a mortgage on the land owned by the contractor that was also subject to the bank's mortgage. The private equity firm promptly recorded its mortgage. In August, after the contractor had received an additional million dollars from the bank, the bank acquired actual knowledge of the private equity firm's mortgage on the land. In October, after the bank had loaned the contractor an additional million dollars, the contractor defaulted on both loans. As a consequence of the default, the bank, pursuant to the terms of the loan, did not advance the final million-dollar installment. At a sale of the land initiated by the bank's foreclosure of its mortgage, to what portion of the sale proceeds does the bank have priority over the private equity firm? Answers: Up to $6 million, the amount the bank initially promised to loan the contractor. Up to $5 million, the amount the bank actually loaned the contractor. Up to $4 million, the amount the bank loaned the contractor before having actual knowledge of the private equity firm's mortgage. Up to $3 million, the amount the bank loaned the contractor before having constructive knowledge of the private equity firm's mortgage.

Answer choice B is correct. The bank's mortgage on the contractor's land secured not only its initial loan of one million dollars, but also future advances the bank made to the contractor. Because the bank was obligated to make bi-monthly loans of one million dollars to the contractor, the bank's mortgage secured these future advances, which eventually totaled five million dollars. The bank has priority over the private equity firm to sale proceeds to the extent the proceeds do not exceed that amount. Answer choice A is incorrect because, although the bank was initially obligated to loan the contractor six million dollars, the bank was not obligated to loan the contractor the final installment of one million dollars due to the contractor's default on its loan obligation and did not do so. Consequently, the bank is entitled to priority over the private equity firm with respect to the five million dollars that the bank did loan to the contractor. Answer choice C is incorrect because, as the bank was obligated to make the bi-monthly loans, the bank is entitled to priority with regard to the loans made by bank after as well as before the bank acquired actual knowledge of the mortgage loan made by the private equity firm. Answer choice D is incorrect because, as the bank was obligated to make the bi-monthly loans, the bank is entitled to priority with regard to the loans made by bank after as well as before the bank had constructive knowledge of the mortgage loan made by the private equity firm as a consequence of the firm's recording of its mortgage.

Question 7341 An individual received a contingent remainder interest in land by will. Subsequently, the individual sought to transfer this interest to his niece via a document. The unsigned document identified the individual as the grantor, the niece as the grantee of the interest, contained an adequate description of the property interest to be conveyed, and expressed an intent to transfer the interest. Does this document operate to transfer the contingent remainder interest to the niece? Answers: No, because the document did not indicate that the transfer was made for good and valuable consideration. No, because the individual did not sign the document. Yes, because a contingent remainder may be transferred during the owner's lifetime. Yes, because the document constitutes a valid deed.

Answer choice B is correct. The grantor's signature is generally required for a document to be a valid deed. Because the individual who held the contingent remainder did not sign the document, it is not a valid deed and will not operate to transfer his contingent remainder interest to his niece. Answer choice A is incorrect because a deed does not require a recitation of consideration. Many deeds effect a gratuitous transfer of property, as is presumably the case in this transfer. Answer choice C is incorrect because, while in most states a contingent remainder may be transferred during the remainderman's lifetime, the document in question is not a valid deed. Answer choice D is incorrect because, as noted with respect to answer choice A, the document is not a valid deed because the grantor did not sign the document.

Question 6418 A small company specializing in providing meditation retreats purchased a tract of land fronting a river in a rural residential area. As the residential area grew around the company's tract of land, the tract became essentially cut off from the only public road. Hoping to avoid the expense of building a bridge across the river, the company approached an owner of one residential plot separating the company's tract from the road to request an easement. The owner, without requiring consideration, granted the company an easement by deed to a strip of land 30 feet wide across his property. The deed did not place any express limitations on the easement's use. The company built a narrow dirt road across the easement to accommodate the limited number of vehicles attending its occasional meditation retreats. A year later, the company began to offer outdoor yoga classes each night. This new service was wildly popular, resulting in increased daily traffic across the easement. On busy evenings, visitors also parked on the dirt road for the duration of their class. After a month of enduring the additional traffic, the owner erected a concrete barrier to prevent any use of the easement to access the company's tract. The company objected, and the owner brought an action to terminate the easement across her property. Should the court allow the owner to terminate the easement on these facts? Answers: Yes, because the company's current use of the dirt road greatly exceeded its initial use of the road. Yes, because the granting of the easement was not supported by consideration and is thus freely revocable. No, because the easement is not terminated by the company's expanded use of the dirt road. No, because the owner, by blocking the dirt road, can no longer seek equitable relief.

Answer choice C is correct. An express easement arises when it is affirmatively created by the parties in a writing that is in compliance with the Statute of Frauds. In addition to any express terms in the written instrument that allow for termination of the easement, an express easement may be terminated by an express written release, merger, severance, abandonment, prescription, or estoppel. Here, although the owner of the servient estate may be able to thwart a change in the scope of an express easement, this change does not constitute grounds for terminating the easement. Answer choice A is incorrect. Although the expanded use of an easement may be enjoined by the owner of the servient estate if such use is not in conformity with the express terms of the easement or is otherwise unreasonable, expanded use is not an independent ground for termination of an easement. Answer choice B is incorrect because an express easement need not be supported by consideration. Once created, the easement cannot unilaterally be revoked by the owner of the servient estate. Answer choice D is incorrect. Although the owner could be sued for interfering with the company's express easement, the proper remedies for this conduct do not include barring the owner from challenging the easement and seeking equitable relief in court.

Question 3148 A buyer entered into a written contract to purchase real property. At closing, the buyer, contending that there was a title defect, refused to pay the seller. The seller brought an action against the buyer to compel the buyer to purchase the property. Which standard should the court apply in determining whether the title is marketable? Answers: The seller must have been unaware of a substantial defect in the title at the time that the contract was entered into. The title must be free of an unreasonable risk of litigation at the time of closing. The title must be free from encumbrances at the time of closing. The title must be free of any defect at the time of closing.

Answer choice B is correct. The marketable title requirement, which is implied in a real property contract unless it is specifically disclaimed, is sometimes characterized as one that protects the purchaser of real estate from buying a lawsuit. Answer choice A is incorrect because, since the marketable title requirement is imposed for the protection of the buyer, a subjective standard based on the seller's knowledge at the time of contracting would only protect the buyer from fraud by a seller. In addition, the marketable title requirement is imposed on the seller at the closing rather than the time of contracting. Answer choice C is incorrect because, although real property must be free from encumbrances for title to be marketable, the test casts a wider net. A property that is free from encumbrances may nevertheless be unmarketable due to a defect in the title itself (e.g., a misnamed grantor or grantee or a variance in the description of the property between different deeds), as the buyer in this case contends. Answer choice D is incorrect because it misstates the standard. This standard is not broad enough because it does not encompass encumbrances. At the same time, it is too broad in that minor defects that do not create a reasonable risk of litigation would not render a title unmarketable.

Question 5879 The owner of a house in a suburban neighborhood converted a driveway from rock to asphalt. The owner's neighbor did not have a driveway but instead parked on the street that ran in front of both of their homes. When asked by the neighbor, the owner told the neighbor that she could also use the driveway to access her house. For 11 years, the neighbor used the driveway on occasion when engaged in tasks such as bringing furniture and appliances into her house, or on days when it was raining heavily. Throughout these years, only the owner maintained the driveway. Recently, the owner sold his house to a couple. The owner informed the couple about the neighbor's use of the driveway when they first looked at the house, but neither the contract of sale nor the deed made reference to it. When the neighbor used the driveway on the first rainy day after the couple moved in, the couple told her that she could no longer use their driveway. The neighbor sued the couple, seeking a judgment that she has a right to use the driveway. In the applicable jurisdiction, the term for the creation of a prescriptive easement is 10 years. Who is likely to succeed? Answers: The couple, because a license, as a property interest, must be in writing. The couple, because the neighbor's license to use the driveway has been revoked. The neighbor, because her use of the driveway created a prescriptive easement. The neighbor, because the couple had notice of her driveway use before buying the house.

Answer choice B is correct. The owner's oral promise to his neighbor constituted a license, which is a non-possessory right to enter the land of another for some delineated purpose. Generally, a license is revoked upon the transfer of the servient estate. Here, the owner sold the house, the servient estate, to the couple. Although a license may not be revoked if detrimentally relied upon, there is no indication that the neighbor expended money or otherwise relied on the owner's promise to her detriment. Consequently, the license that the owner had granted to the neighbor to use the driveway has been revoked. Answer choice A is incorrect because a license, unlike an easement, may be created orally. Answer choice C is incorrect. Although it is arguable that the neighbor's use of the driveway was sufficient to permit the creation of a prescriptive easement, and it is clear that the neighbor's use of the driveway exceeded the 10-year period required by law, the neighbor's use of the driveway did not create a prescriptive easement because it was done with the owner's permission. Answer choice D is incorrect because the couple's notice of the neighbor's use of the driveway prior to entering into a contract to purchase the owner's residence does not bind them to permit that use to continue.

Question 6422 By statute, a jurisdiction provides: "Any judgment properly filed shall, for ten years from filing, be a lien on the real property then owned or subsequently acquired by any person against whom the judgment is rendered." In addition, the recording act of the jurisdiction reads, in its entirety, as follows: "No conveyance or mortgage of real property shall be good against subsequent purchasers for value and without notice unless the same be recorded according to law." This act has been interpreted as not providing any grace period for recording a conveyance or mortgage. An owner conveyed land by a warranty deed to his adult child. The child recorded the deed a week later. Three days after the conveyance to the child and without knowledge of it, a creditor of the owner properly filed a judgment against the owner. The creditor then filed suit against the owner and his child to foreclose the judgment lien against the land. If the court rules against the creditor, which of the following is the most likely reason? Answers: The owner's warranty of title to his child protects the land from the creditor's claim. The creditor is not a purchaser for value. The warranty deed has priority over any valid judgment lien on the land. The child's week-long delay in recording the deed was not unreasonable.

Answer choice B is correct. The recording act protects a subsequent purchaser for value of real property from prior unrecorded conveyances or mortgages of that property of which the subsequent purchaser did not have knowledge. At the time that the creditor properly filed a judgment against the owner, thereby creating a judgment lien against real property owned or subsequently acquired by the owner, the conveyance of the land from the owner to the child had not been recorded and the creditor had no knowledge of this conveyance. Thus, the creditor would be protected by the recording act, but only if the creditor was treated as a purchaser for value. Because the court ruled against the creditor, the jurisdiction, like a majority of jurisdictions, must not treat a lien creditor as a purchaser for value. Answer choice A is incorrect because the owner's warranty of title, as part of a warranty deed, runs to his child, as grantee. Although the owner may be obligated to defend the child's interest in the land against a challenge by the creditor as a consequence of this warranty of title, the warranty has no effect on the rights of the creditor to the land. Answer choice C is an incorrect statement of the law. A judgment lien on real property has priority over a subsequent conveyance of real property, even when that conveyance is by a warranty deed. Answer choice D is incorrect because the recording act provides no grace period for a reasonable delay in recording a deed. Thus, the fact that the child's delay in recording the conveyance was likely reasonable would not result in a judgment for the child and against the creditor.

Question 7512 A homeowner whose house sat on an irregularly shaped lot constructed a storage shed that by mistake rested entirely on his neighbor's property. The storage shed stood for 16 years before the neighbor discovered the mistake when selling her property. The homeowner apologized and dismantled the shed. The neighbor then transferred title to her property to a buyer who promptly recorded the deed. Six months later, the homeowner died, devising all of his real property interests to his son. When the son sought to construct a storage shed in the same location as the prior shed, the buyer objected and initiated a lawsuit against the son. In the applicable jurisdiction, the statute of limitations for adverse possession is 15 years and the recording act is a race type recording act. Which of the following is the buyer's best argument that the land on which the original storage shed was located belongs to the buyer? Answers: The buyer is entitled to this land under the recording act. The buyer is the owner of the land pursuant to the doctrine of estoppel by deed. The homeowner's removal of the shed returned not only possession but also ownership of this land to the neighbor. The homeower's possession of this land was not hostile.

Answer choice D is correct. Among the requirements for adverse possession is that the adverse possession must be hostile. In a majority of jurisdictions, "hostile" merely means without the owner's permission; it does not require that the possessor purposefully seeks to defeat the owner's title because the intent of the possessor is irrelevant. In jurisdictions that consider intent, some will grant title to a possessor who, in good faith, thought he had the legal right to possess (i.e., believed that the property was not owned or thought that he owned the property). Others require the intent to be based on bad faith (i.e., aggressive trespass). Here, because the homeowner did not act in bad faith, and the other answer choices do not defeat the homeowner's adverse possession of the land, this is the buyer's best argument. Answer choice A is incorrect. Under a race recording act, a purchaser of property who records his property interest before the competing claimant has priority over that claimant, regardless of whether the purchaser has notice of the other claim at the time of purchase. Although a buyer is generally protected under this type of recording act from any person whose property rights arose prior to the buyer's acquisition of the property, no type of recording act protects a subsequent taker for value from a property interest acquired by operation of law, such as property acquired by adverse possession. Since generally such ownership arises automatically upon the expiration of the statutory period, there is no documentation to be recorded. Answer choice B is incorrect. The doctrine of estoppel by deed prevents a grantor who does not own property at the time that the grantor purports to transfer ownership of it to the grantee from asserting ownership of that property when the grantor subsequently acquires ownership of the property. Here, this doctrine does not apply because the homeowner did not deed the property in question to the buyer. Answer choice C is incorrect because, once the homeowner's right to this property arose because of adverse possession, the homeowner cannot return ownership of this property orally or by simply removing the trespassing structure. The homeowner must transfer ownership of the adversely possessed property by deed.

Question 6217 A winemaker had been in the wine making business for many years, and decided to retire and sell the winery. When the winemaker's son, who was an alcoholic and estranged from his family, found out about the sale, he approached the winemaker and told him that he wanted to run the winery. After some discussion, the winemaker told his son that if he remained sober for the next 12 months, he would convey the winery to him. The winemaker, convinced of his son's sincerity, executed a valid deed and gave it to his son. At the end of the twelve months, the son, who had begun consuming alcohol again, tried to take possession of the winery, but the winemaker refused to surrender possession. The son then recorded the deed. The winemaker has filed an action to set aside the conveyance. Who will prevail? Answers: The son, because the son recorded the deed. The son, because the oral condition is not enforceable. The winemaker, because the son's promise could be performed within one year. he winemaker, because he intended to convey the land only if his son remained sober.

Answer choice B is correct. Transfer of a deed to the grantee creates a presumption that the grantor intended to make a present transfer of the property interest. Parol evidence is admissible to show that the grantor lacks such intent. However, when the grantor transfers the deed to the grantee subject to a condition that does not appear in the deed (e.g., an oral condition), parol evidence is not admissible and the condition is not enforceable. Here, the sobriety condition was not included in the deed, so it is not enforceable. Therefore, the son will prevail. Answer choice A is incorrect because, although the recording of a deed by a grantor is evidence that the grantor had the present intent to transfer the property to the grantee, the recording of a deed by the grantee does not affect the enforceability of an oral condition placed on the transfer by the grantor. Answer choice C is incorrect because, although the son's promise could be performed within one year and therefore need not be in writing to be enforceable for that reason, a contract for the transfer of real property must be writing. Here, the son's promise to remain sober for one year in exchange for his father's promise to transfer ownership of the winery to the son was not in writing. Answer choice D is incorrect because the winemaker had the present intent to convey the winery to his son, and he also intended for the conveyance to be conditioned upon his remaining sober. However, because the oral condition was not included in the deed, it is not enforceable.

Question 3119 A woman obtained ownership of a cottage after her grandmother died. The owner, who lived out of state, decided to sell the cottage to a buyer. The parties agreed that closing would occur the following week. The owner had casualty insurance on the cottage, while the buyer did not obtain such insurance. On the evening before the closing was to take place, a fire caused by lightning destroyed the cottage. The jurisdiction has not adopted the Uniform Vendor and Purchaser Risk Act. If the seller seeks specific performance, may the buyer rescind the contract for the sale of the cottage? Answers: No, because the buyer bears the risk of loss. No, but the owner must give the buyer credit against the purchase price in the amount of the insurance proceeds. Yes, because the owner has casualty insurance and can more easily bear the loss. Yes, but the buyer must reimburse the owner for the cost of the casualty insurance.

Answer choice B is correct. Under the doctrine of equitable conversion, equitable title to real property passes to the buyer upon entering the contract, even though the seller retains legal title. Most jurisdictions place the risk of loss between the contract and the closing on the buyer. However, if the seller has casualty insurance on the property, the seller must give the buyer credit in the amount of insurance proceeds against the purchase price. Answer choice A is incorrect because it is incomplete; although the buyer does bear the risk of loss, if the seller has insurance, the buyer will not be liable for the full amount. Answer choice C is incorrect because the fact that the seller has casualty insurance does not mean that the seller has to bear the loss entirely. Answer choice D is incorrect because it misstates the law; the buyer is under no duty to reimburse the seller for the cost of the insurance; the buyer must proceed with the sale, minus any insurance proceeds received by the seller.

Question 6432 A buyer entered into a written contract to purchase real property from its owner. The buyer asked that the owner convey the property to the buyer and her brother as tenants in common. The owner noted that the buyer's brother would need to attend the closing to sign the necessary paperwork. Because the brother lived in another state and could not attend the closing, the buyer brought her roommate to the closing instead. The roommate pretended to be the buyer's brother and signed all the necessary paperwork with the brother's name. The buyer paid the full purchase price, and the deed granting the buyer and her brother half interests as co-tenants was recorded on the same day. Unknown to any of the parties, the evening before the closing, the buyer's brother had died in a car accident. The brother's valid probated will devised all of his property to his wife. The brother's wife has brought an action against the buyer, who has taken sole possession of the property, and the original owner to quiet legal title to an undivided one-half interest in the property. Who should the court find has legal title to the real property, and in what proportions? Answers: The original owner. The buyer. One-half in the buyer and one-half in the original owner. One-half in the buyer and one-half in the brother's wife.

Answer choice C is correct. A deed to a nonexistent grantee is void as to the nonexistent grantee. Therefore, the purported conveyance of a one-half interest in the real property to the buyer's brother failed, and the original owner retains this one-half interest. The conveyance of a one-half interest in the property to the buyer is unaffected. (Note: The buyer should be successful in an unjust enrichment action to recoup one-half of the purchase price from the original owner.) Answer choice A is incorrect because the deed, although void as to the buyer's brother, is still valid as to the buyer. Answer choice B is incorrect because the buyer was only granted a one-half interest in the real property. The remaining one-half interest granted to her brother by the deed does not go to her just because she paid the purchase price, although she would have an action against the original owner for unjust enrichment. Answer choice D is incorrect. When one of the contracting parties dies prior to the performance date of the contract, the person entitled to the buyer-decedent's real property can compel the transfer of the property to herself. However, here, the brother was not the buyer because he was never actually a party to the contract and never authorized the buyer's roommate to act as an agent on his behalf in the transaction. Consequently, the one-half interest deeded to the buyer remains with the original owner.

Question 1577 Ten years ago, a seller sold land to a buyer, who financed the purchase price with a loan from a bank that was secured by a mortgage on the land. The buyer purchased a title insurance policy running to both the buyer and the bank, showing no liens on the property other than the buyer's mortgage to the bank. Eight years ago, the buyer paid the mortgage in full. Seven years ago, the buyer sold the land to an investor by a full covenant and warranty deed without exceptions. Six years ago, the investor gave the land to a donee by a quitclaim deed. Last year, the donee discovered an outstanding mortgage on the land that predated all of these conveyances. As a result of a title examiner's negligence, this mortgage was not disclosed in the title insurance policy issued to the buyer and the bank. Following this discovery, the donee successfully sued the buyer to recover the amount of the outstanding mortgage. If the buyer sues the title insurance company to recover the amount he paid to the donee, is he likely to prevail? Answers: No, because the buyer conveyed the land to an investor. No, because the title insurance policy lapsed when the buyer paid off the bank's mortgage. Yes, because the buyer is protected by the title insurance policy even though he no longer owns the land. Yes, because the buyer was successfully sued by a donee and not by a bona fide purchaser for value.

Answer choice C is correct. Although the lender's policy of title insurance ended when the loan was repaid, an owner's policy of title insurance continues to protect the owner if the owner (here, the buyer) is ever successfully sued on a title covenant in a future conveyance. The donee successfully sued the buyer on the title covenant in the buyer's full warranty deed which was given without exception to the investor. The buyer can now recover on the owner's policy of title insurance. For these reasons, answer choices A and B are incorrect. Answer choice D is incorrect. Although this response correctly concludes that the buyer will prevail against the title insurance company, it misstates the rationale. It does not matter whether the buyer was sued by a donee or a bona fide purchaser for value.

Question 4216 A shopkeeper purchased the building that houses her shop, incurring an obligation equal to the fair market value of the building, plus interest to the owner of the building. The obligation is evidenced by a promissory note and secured by a mortgage on the building. Several years later, the shopkeeper sold her business and the building to a third party, who assumed liability for the obligation. At the request of the third party and without the knowledge of the shopkeeper, the original owner of the building agreed to release the mortgage. Subsequently, the third party defaulted on the obligation. The current fair market value of the building exceeds the amount of the unpaid obligation. The original owner of the building sued the shopkeeper for the amount of the unpaid obligation. Can the original owner recover this amount from the shopkeeper? Answers: Yes, because the shopkeeper remains personally liable for the unpaid obligation. Yes, because the obligation was incurred in conjunction with a purchase money mortgage. No, because the release of the mortgage discharged the shopkeeper's personal liability. No, because the third party assumed liability for the obligation.

Answer choice C is correct. Although the shopkeeper/mortgagor is personally liable on the loan made by the original owner of the building (i.e., mortgagee), the building owner's release of the related mortgage discharges that liability; thus, answer choice A is incorrect. Answer choice B is incorrect because the fact that the mortgage, which was given in conjunction with the obligation, was a purchase money mortgage is irrelevant to the issue of the personal liability of the mortgagor. Answer choice D is incorrect because, although the third party assumed liability for the loan obligation, this assumption did not relieve the shopkeeper of personal liability for the same obligation.

Question 3143 A widow held a life estate in a house and several acres of land in a semi-rural area. She lived in the house and harvested berries from the numerous wild berry bushes on the property each June. The widow personally consumed the berries or gave them away to family and friends, but did not sell them to third parties. One May, just before the berries were to be harvested, the widow died. The widow's children, her heirs, sought to enter the land to harvest the berries, but the remainderman, the new owner, objected, claiming that he had sole right to the berries Do the heirs have the right to return and harvest the berries? Answers: Yes, because the berries were the widow's personal property. Yes, because the widow would have been able to harvest them had she survived. No, because the berries grew wild. No, because the widow did not sell the berries to third parties.

Answer choice C is correct. Fructus naturales are wild crops that are not cultivated; such crops are considered real property and pass automatically with the land. Here, title to the land, including the unharvested berries, reverted to the remainderman at the time of the widow's death. Accordingly, the remainderman has the sole right to the berries. Had the berries been fructus industriales, that is, had the widow purposely planted and cultivated the berries, then they would have been considered personalty. Answer choice A is incorrect because the berries were not yet the widow's personal property; they would have become her personal property if she had already harvested them, and they would have been considered personalty had she cultivated them. However, because they grew wild, they pass with the land. Answer choice B is incorrect because, while it is true that she would have been able to harvest them had she survived, the land, including the berries, passed to the remainderman at the time of her death. Answer choice D is incorrect because what the widow did with the berries is irrelevant to the question of whether her heirs are entitled to enter the land and harvest them.

Question 4219 A bank provided a loan to the purchaser of a parcel of undeveloped land. As security for the loan, the purchaser granted the bank a mortgage with respect to the parcel of land. Subsequently, the purchaser subdivided the land into three lots. The purchaser sold two of the three lots, each to a different buyer at a different time. Each buyer obtained a loan to finance the purchase from a different bank, neither of which was the bank that provided the original loan to the purchaser. Each buyer gave his lender a mortgage on his lot. The purchaser later defaulted on his bank loan. The bank filed an action to foreclose on all three parcels of land. The buyers of the two lots each filed an appropriate motion to protect their interests to the maximum extent possible. Which of the following best describes the likely outcome of the bank's attempt to proceed against the three lots? Answers: The bank may proceed against any of the three lots in any order that it chooses. The bank must proceed first against the lot retained by the purchaser and then against the lots in the order in which the mortgages on them were created. The bank must proceed first against the lot retained by the purchaser and then against the lots in the inverse order in which the mortgages on them were created. The bank must proceed against all of the lots in a manner that recovers an equal amount from each.

Answer choice C is correct. Generally, upon default of an obligation, a mortgagee may foreclose on any and all parcels of real property of the mortgagor that serve as security for the obligation. If there are junior security interests with respect to some of those parcels of real property, however, those junior interests may petition the court for protection of their interests under the doctrine of marshaling of assets. Under this doctrine, the holder of a senior security interest must first proceed against the property on which there are not any junior security interests, and then against the property on which the junior interest was more recently created, before proceeding against property on which the junior interest was more remotely created. Answer choice A is incorrect because, although a mortgagee may generally proceed with a foreclosure against any real property of the mortgagor that serves as security for the obligation, in this case the doctrine of marshalling of assets applies to prevent the mortgagee from exercising that discretion. Answer choice B is incorrect because the doctrine of marshalling of assets requires that the holder of the senior security interest proceed first against the property on which the junior interest was more recently created. Answer choice D is incorrect because the holder of the senior security interest must proceed first against property on which there are not any junior security interests, and then against the property on which the junior interest was more recently created, before proceeding against property on which the junior interest was more remotely created.

Question 3155 After inheriting a substantial amount of money, a man purchased a large estate in the mountains adjacent to a ski resort intending to operate the estate as a seasonal rental property used exclusively to generate rental income. The man made the purchase with cash. Unable to properly manage his wealth, he was impoverished a few months later. He therefore procured a mortgage on the estate from a credit union, and the mortgage was properly executed and recorded. In light of a struggling economy, credit union executives were confident the man would default on the loan and wanted to ensure the estate was properly maintained in anticipation of a subsequent sale. The credit union therefore sought to obtain a court order confirming its right to possession of the estate in order to make repairs and prevent further deterioration of the property. Could the credit union take possession of the estate? Answers: No, in a title theory state, absent default by the mortgagor. No, in a lien theory state, unless the mortgage included an acceleration clause. Yes, in a title theory state, until the mortgage has been fully satisfied. Yes, in a lien theory state, because the mortgagee is considered the owner of the land during the term of the mortgage.

Answer choice C is correct. In a title theory state, legal title is in the mortgagee (here, the credit union) until the mortgage has been fully satisfied. Thus, the mortgagee is theoretically entitled to take possession at any time, although the mortgagee is typically prohibited by the terms of the mortgage from taking possession of the property before default occurs. As such, the credit union can make repairs, take rent, prevent waste, and lease out vacant space. Answer choice A is incorrect because in a title theory state, the mortgagee has the right to possession even if the mortgagor has not yet defaulted. Answer choice B is incorrect. It is true that the mortgagee cannot take possession prior to foreclosure in a lien theory state because the mortgagor is considered to be the owner of the land until foreclosure. However, an acceleration clause would not change that outcome here. An acceleration clause provides that the full amount of the mortgage obligation becomes due upon default. It does not change what party has a right to possess the mortgaged property. Answer choice D is incorrect because in a lien theory state, the mortgagee cannot take possession prior to foreclosure because the mortgagor is considered to be the owner of the land until foreclosure.

Question 5207 A corporation purchased a building with the proceeds from a bank loan and granted the bank a mortgage on the building as security for the loan. Several years later, the corporation granted a second mortgage on the building to a private lender in exchange for a much smaller loan. Currently, the corporation is in compliance with its obligations with respect to the bank loan, but has defaulted on the smaller private loan. The private lender intends to initiate a foreclosure action against the corporation. Must the private lender join the bank as a necessary party to this action? Answers: Yes, because the bank's mortgage is the senior mortgage. Yes, because the bank's mortgage is a purchase-money mortgage. No, because the bank's mortgage is not affected by the private lender's foreclosure action. No, because the private lender's loan is less than the bank's loan.

Answer choice C is correct. The bank's mortgage interest in the building has priority over the private lender's mortgage because it was first in time and because it was a purchase-money mortgage. As the senior mortgage, it is not affected by the foreclosure of the private lender's junior mortgage. Consequently, the private lender need not join the bank as a necessary party to the private lender's foreclosure action. Answer choice A is incorrect because, while it is true that the bank's mortgage is the senior mortgage because it was first in time, as the senior mortgage, it is not affected by the foreclosure of the private lender's junior mortgage. Consequently, the private lender need not join the bank as a necessary party to the private lender's foreclosure action. Answer choice B is incorrect. While the bank's mortgage is a purchase-money mortgage because the proceeds from the loan that it secures were used to purchase the building, there is no special requirement that requires the joinder of the holder of a purchase-money mortgage when a junior mortgage is foreclosed. Answer choice D is incorrect because the fact that the private lender's loan is less than the bank's loan is irrelevant as to whether there is a need to join the bank as necessary party to the private lender's foreclosure action. What matters is that the bank's mortgage interest is senior to the private lender's mortgage interest and not affected by the foreclosure of the private lender's junior mortgage.

Question 6431 An owner of two adjoining farms conveyed one of the farms to a farmer in fee simple by a warranty deed. The deed contained the following provision: "Grantor, his heirs and assigns agree to reimburse grantee, his heirs and assigns, one-half the cost of any reasonable expense incurred as the result of having to repair the stone wall situated on the conveyed property at its common boundary with property retained by grantor. The parties intend a covenant running with the land." The farmer never recorded the deed. A year later the farmer conveyed the farm to a businessman in fee simple by warranty deed, and thirty years after that, the businessman conveyed the farm in fee simple by warranty deed to a horse breeder. Neither of these deeds referenced the stone wall or the original covenant, but both were properly and promptly recorded. The horse breeder, after investigating the past deeds to his farm and discovering the original deed to the farmer, spent $2,000 to raise the height of the stone wall. He then demanded that the owner's son, who had inherited his farm from the owner, reimburse him for $1,000. The son refused, and the horse breeder brought an appropriate action to recover the sum. The statutory period in the jurisdiction for acquiring title through adverse possession is ten years. If the court rules for the owner's son in the horse breeder's action, which of the following is the most likely reason? Answers: The farmer's deed from the owner containing the covenant was not recorded. The horse breeder's claim is precluded by the doctrine of adverse possession. The horse breeder's expenditures were not incurred for a reasonable repair. The son inherited his land, negating vertical privity.

Answer choice C is correct. The covenant in the original deed from the owner to the farmer provides that only reasonable expenses will be reimbursed. Because all of the requirements for a covenant to run with the land are met, the only reason provided that might justify a finding for the son would be a finding that raising the height of the wall is not a reasonable repair covered by the covenant. Answer choice A is incorrect because the son was not a subsequent purchaser of the land protected under the recording statute. Instead, he inherited title to the land, along with the obligations under the covenant that ran with the land. Answer choice B is incorrect. Because the wall is on the horse breeder's land, not on the son's land, adverse possession does not apply. Answer choice D is incorrect because the burden of the covenant runs if the successor holds an estate of the same duration as the original party. Vertical privity does not require that the current owner of the burdened property acquire it by purchase.

Question 5867 The sole, unmarried owner of a residence died. She validly devised the residence to her long-term companion with whom she had lived for over 20 years. The residence was devised to the companion "for life or until she vacates the premises, and then to my nephew." Several years after the owner's death, the nephew transferred by quitclaim deed "any interest I have" in the residence to a creditor in satisfaction of a debt that the nephew had incurred. The deceased owner's companion continues to live in the residence. Which of the following most accurately describes the creditor's interest in the residence? Answers: The creditor has a vested remainder in the residence. The creditor has an executory interest in the residence. The creditor has both a vested remainder and an executory interest in the residence. The creditor has a mere expectancy with regard to the residence until the companion dies or vacates the premises.

Answer choice C is correct. The deceased owner's companion has a determinable life estate. The nephew had two future interests in the residence. The nephew held a vested remainder in the residence that will be transformed upon the companion's death into a fee simple absolute. A vested remainder interest is transferrable during the holder's life as well as upon death. The nephew also held a shifting executory interest that will be transformed into a fee simple absolute in the event that the companion vacates the premises prior to her death. In most jurisdictions, an executory interest may be transferred during the holder's life as well as upon her death. Consequently, the nephew's creditor now holds both future interests, the vested remainder and the executory interest, in the residence. Note that neither the fact that the transfer was made by a quitclaim deed nor the fact that the transfer was made in satisfaction of antecedent debt affects the creditor's interests in the property. Answer choice A is incorrect because, although the nephew did have a vested remainder in the residence, which he transferred to the creditor during his lifetime, the nephew also had an executory interest in the residence, which he also transferred to the creditor. Answer choice B is incorrect because the nephew had an executory interest in the residence, which he transferred to the creditor during his lifetime. The nephew also had a vested remainder in the residence, which he also transferred to the creditor. Answer choice D is incorrect. An expectancy is not a true property interest, only a right that will not come into existence until the happening of an event. For example, an heir is said to have an expectancy of an inheritance, but the heir has no legal right to any property until the death of the property owner to whom the heir is related. Here, the creditor holds two future legal property interests in the residence.

Question 5872 A rancher subdivided a portion of his ranch that had recently been annexed by the city into 30 two-acre lots. The rancher filed a subdivision plan that restricted the use of each lot to one single-family residence, but placed no other restrictions on the lots. The rancher then sold a lot to a speculator. The deed contained the single-family residential restriction, but no other restrictions. Immediately thereafter, the housing market in the area plummeted and the rancher was unable to sell any of the remaining lots for almost three years. During the next two-year period, the rancher sold 28 of the remaining 29 lots. At the time that the portion of the ranch was annexed by the city, the city's building code prohibited a single-family residence of more than two stories. After the rancher sold the first lot to the speculator, but before he sold any of the remaining lots, the city, acting in response to complaints by developers, modified its building code to permit three-story single-family residences. However, the rancher did not want any of the residences built on the lots to be as high as his own three-story residence located on the ranch. Thus, he included in the deeds to each of the 28 lots that he sold after the market rebounded a two-story height restriction for each single-family residence. The rancher made the speculator aware of this height restriction by letter, but the speculator did not respond. Subsequently, the speculator sold his lot to a couple. The deed to the couple contained the single-family residence restriction, but made no mention of the two-story height limitation. The rancher, upon learning of the couple's plans to construct a three-story family residence, has filed suit on his own behalf, as well as the owners of the other 28 lots, seeking an injunction to prevent the couple from building a three-story family residence. All of the 28 owners of the other lots who have built residences on their lots have not violated the two-story height restriction. For whom is the court likely to rule? Answers: The rancher, because the common scheme of the two-story height restriction was readily apparent to the couple. The rancher, because he made the speculator aware of the height restriction before the speculator sold his lot to the couple. The couple, because the two-story height restriction was not contained in their deed, the speculator's deed, or the subdivision plan. The couple, because the city building code takes priority over a private restrictive covenant.

Answer choice C is correct. The two-story height restriction was not contained in the couple's deed, the speculator's deed, or the recorded subdivision plan. Therefore, this restriction cannot be imposed on the couple's use of the lot as either a real covenant or an express equitable servitude. Answer choice A is incorrect because the common scheme did not exist at the time that the rancher transferred the first lot to the speculator, but was developed by the rancher in response to the change in the city building code. Even if the couple was aware of the common scheme from the fact that there were no three-story residences on any of the other developed lots, this awareness does not bind them as owners of their lot to that common scheme because the rancher failed to manifest an intent to bind their lot to that scheme at the time that he sold it to the speculator. Had the rancher evidenced his intent to impose this restriction on all thirty lots prior to the sale of the lot to the speculator, such as by including a provision in the subdivision plan, then the speculator and the couple might have been subject to the restriction as an implied reciprocal servitude. Answer choice B is incorrect because, as with the couple's awareness of the common scheme, the speculator's knowledge of the two-story height restriction did not arise until after he had become the owner of the lot. Moreover, the height restriction was not included in either deed or the subdivision plan. Answer choice D is incorrect. Although the couple is subject to the city's building codes, if they were also subject to the two-story height restriction found in the deeds of the 28 lots (which they are not), then the private restriction could still be enforced. The owners of private property may subject their property to greater restrictions on the use of their property than are imposed by a governmental entity.

Question 6225 A fitness company entered into a ten-year lease with the landlord of a gym facility. The lease required the fitness company to maintain the gym equipment in proper, working condition, and to upgrade or replace any of the equipment as required by the safety guidelines for gymnasiums issued by a national organization of gymnasiums. In addition, the lease specified that all of the fitness company's clients must sign a valid waiver releasing the current landlord from liability for any injury arising from their improper use of the gym equipment. One year into the lease, the landlord transferred the remaining term of the fitness company's lease to a large fitness conglomerate. The transfer occurred without the fitness company's consent. The fitness company paid rent to the conglomerate, but they stopped making their clients sign the liability waiver because the conglomerate did not require any of their gym members to sign one. The conglomerate has brought an action against the fitness company to enforce this covenant in the lease. Who will prevail? Answers: The fitness company, because the conglomerate does not require liability waivers from its members. The fitness company, because they did not consent to the assignment of the gym facility. The conglomerate, because the liability waiver requirement touches and concerns the land. The conglomerate, because the fitness company had required its clients to sign the waiver in the past.

Answer choice C is correct. Under the doctrine of attornment, the tenant is bound to honor any covenant in his lease that has been assigned by the landlord to a third party, if the covenant touches and concerns the land. Here, the liability waiver touches and concerns the land because it has to do with clients using gym equipment in the gym facility. Answer choice A is incorrect. Although the conglomerate does not require liability waivers from its members, the covenant in the lease requires such liability waivers by clients of the fitness company in using the gym facility. Answer choice B is incorrect because the landlord can assign his ownership interest to a third party without the tenant's consent. Answer choice D is incorrect. The fact that the fitness company had required its clients to sign the waiver in the past does not make the covenant enforceable by the conglomerate. The liability waiver covenant must touch and concern the land in order to be enforceable.

Question 2014 A buyer entered into a contract to purchase a 20-year-old residence from its current owner and occupant. Among the terms of the contract was a warranty that the residence was free from termite infestation and damage and a requirement that the seller obtain a termite inspection and provide the buyer with a report of the inspection. The seller timely complied with this requirement. In the report, the inspector indicated that no evidence of termites was found, but noted that there were certain areas of the house that the inspector was unable to access due to existing structures, such as interior walls and ceilings. At closing, the seller provided the buyer with a deed that the buyer promptly and properly recorded. A few weeks later, the buyer, in the process of remodeling the residence, uncovered live termites and extensive damage in the wooden beams that supported the walls and floor of the house. The seller had not been aware of the presence of the termites in the house or the damage that they had done to its structure. The buyer sued the seller for breach of the warranty relating to termites. Who will prevail? Answers: The buyer, because the seller breached the warranty of fitness or suitability. The buyer, because the buyer could not have learned about the existence of the termites or their damage to the residence through a reasonable pre-sale inspection. The seller, because promises in the contract merged into the deed. The seller, because the seller complied with the terms of the contractual warranty.

Answer choice C is correct. Under the doctrine of merger, a buyer of property generally cannot sue on promises contained in the contract, such as a termite warranty, once closing has taken place. Answer choice A is incorrect because, while the warranty of fitness or suitability does apply to the sale of a residence, it only applies to a newly constructed residence and is imposed only on a commercial seller, typically the builder or developer of the residence. Here, the residence was neither newly constructed nor was the home owner a commercial seller. Answer choice B is incorrect because the express warranty given by the seller does not require, unlike the implied warranty of fitness or suitability, the buyer to lack the ability to uncover the presence of termites or their damage of the residence. Answer choice D is incorrect because, despite the seller's lack of knowledge about the termites, the seller breached the contractual warranty by failing to deliver a termite-free home. However, because the contract merged into the deed, the seller would prevail here whether or not she even made the attempt to comply with the provision, had the buyer still accepted the deed.

Question 5203 An auto mechanic borrowed money to buy an auto repair garage. As security for repayment of the loan, the mechanic transferred ownership of the garage to a title company pursuant to a deed of trust. Under the terms of the deed of trust, upon notification by the lender that the auto mechanic had defaulted on the loan, the title company was authorized to transfer title to the lender if after three months the auto mechanic had not redeemed the property. This jurisdiction recognizes a deed of trust as an acceptable form for a security interest in real property. One year later, the auto mechanic defaulted on the loan. The lender notified the title company of the default. After three months, the title company transferred ownership of the garage to the lender pursuant to the terms of the deed of trust. The following month, the auto mechanic attempted to repay the full amount of the loan obligation. The lender rejected this payment. The auto mechanic has filed suit to regain title to the auto repair garage. How should the court rule? Answers: In favor of the lender, because the auto mechanic did not redeem the garage within three months after the lender notified the title company. In favor of the lender, because a deed may be transferred in lieu of foreclosure. In favor of the auto mechanic, because the clause in the deed that limited the time to redeem the garage is invalid. In favor of the auto mechanic, because he used the money from the loan to purchase the garage.

Answer choice C is correct. Until property that is subject to a mortgage or a deed of trust is foreclosed upon by sale, the borrower has a right to redeem the property by satisfying the obligation for which the property serves as security. While this right may be waived after the borrower has defaulted, the terms of the mortgage or deed of trust cannot waive this right as a matter of course. Consequently, because the clause in the deed of trust in question is invalid and the court should permit the auto mechanic to regain title to the garage by paying the loan obligation in full. Answer choice A is incorrect because, although the auto mechanic did not comply with the terms of the deed of trust, the term that required redemption of the garage within three month was invalid as a clog on his equitable right to redeem the property prior to a foreclosure sale. Answer choice B is incorrect because, while after default a borrower may agree to transfer property that served as security for a loan to the lender in lieu of foreclosure, here the provision for the transfer of the deed in lieu of foreclosure was contained in the deed of trust itself. Therefore, it constitutes an unenforceable clog on the mechanic's equitable right of redemption. Answer choice D is incorrect because the equitable right of redemption is not limited to purchase-money mortgages or deeds of trust.

Question 5200 A sporting goods store owner purchased football equipment from a supplier and gave the supplier an unsecured, nonnegotiable promissory note. Prior to the date on which payment of the note was required, the owner asked the supplier for an extension of time to pay the note. The supplier demanded in exchange that the owner execute a mortgage with respect to the store itself to serve as security for payment of the note. When the owner refused, the supplier threatened to reveal to the police that the owner was running an illegal bookmaking operation in the back of the store. Acting under duress, the owner executed the mortgage, which the supplier promptly recorded. Shortly afterwards, the owner transferred ownership of the store to his daughter as a gift. The daughter assumed the obligation to pay the promissory note. The note is now in default and the supplier has instituted foreclosure proceedings on the mortgage. Can the daughter assert the owner's defense of duress against the supplier? Answers: No, because the supplier recorded its mortgage prior to the transfer of the store to the owner's daughter. No, because the daughter received ownership of the store as a gift. Yes, because the daughter is related to the owner. Yes, because, as a donee, the owner's daughter is entitled to enjoy the same rights with respect to the store as those enjoyed by the owner.

Answer choice D is correct. A donee who takes property that has been mortgaged is entitled to assert the donor's defenses against the mortgagee. (Note: By contrast, the purchaser of a property that has been mortgaged who assumes the mortgage cannot generally assert the defenses of mortgagor-seller against the enforcement of the mortgage if the purchase price reflects the assumption of the mortgage. Otherwise, the purchaser would, by being permitted to avoid the assumed obligation, be unjustly enriched.) Answer choice A is incorrect. While the supplier's recording of the mortgage serves to protect the supplier from any claim by a subsequent good faith purchaser that the purchaser's rights have priority over the supplier's mortgage, the owner's daughter took ownership by gift rather than by purchase. Consequently, the supplier's recording of its mortgage is not relevant to the ability of the daughter to assert her father's defense. Answer choice B is incorrect because, as noted with regard to answer choice D, it is the fact that the daughter took ownership of the store through a gift that allows her to assert her father's duress defense. Answer choice C is incorrect because the familial relationship of the daughter to the owner is not relevant. Her ability to assert the owner's defense of duress arises from her status as donee of his property interest in the mortgaged store and not from her status as the owner's daughter.

Question 4386 A landlord, the owner of the only shopping center in a small town, and a tenant, a new small-business owner, entered into a lease for a commercial shopping space in the shopping center. The tenant was unsure that the community would support her new business, so she wanted to limit the terms of the lease to a maximum of two years. The landlord, however, insisted on an at-will tenancy for a minimum of ten years, and included the following clause in the lease: "pursuant to this Lease, Landlord is given the express right to terminate the leasehold with Tenant by giving 30 days' notice." The lease omitted language giving the tenant a similar termination right. Due to the lack of commercial space available to rent in the area, the tenant agreed. Six months into the lease, the tenant terminated the lease in writing with 30 days' notice, explaining that, although sales at the shopping space were technically covering all of the tenant's expenses, the tenant had found lower rent in a nearby town, which she believed would be a more successful market. The landlord sued the tenant for breach of the lease. Will the landlord prevail in the breach of lease action? Answers: Yes, because the lease contract reserved the right of termination only for the landlord. Yes, because the tenant's reasons for terminating the lease were in bad faith. No, because termination rights in at-will tenancies cannot be limited. No, because the lease's unconscionability gave the tenant the right to terminate the lease.

Answer choice D is correct. A tenancy at will is a leasehold estate that may be terminated by either the landlord or the tenant. If only one party is expressly given the right to terminate the leasehold, the lease may, when taking into account all other circumstances, be unconscionable. In such a case, either party is given the ability to terminate the lease. In this case, the landlord—being the owner of the only commercial shopping center in town—had such superior bargaining power that enforcing the terms of the lease against the tenant would be unconscionable. Accordingly, both parties had the right to terminate this lease. Answer choice A is incorrect because an at-will tenancy terminable by only one party is often unconscionable, and if so, the tenant's right of termination exists by implication. Answer choice B is incorrect because good faith is irrelevant in determining whether a party validly terminated an at-will tenancy; here, the tenant gave proper notice of termination. Answer choice C is incorrect because termination rights in at-will tenancies can be limited; for example, a landlord is not granted an implied right of termination when a lease expressly gives a tenant the right of termination but is silent as to the landlord.

Question 5891 A seller owned a multi-unit residential building that he primarily rented to friends and family. The seller regularly hosted social gatherings on the rooftop deck until he decided to sell the building. Before accepting offers on the building, the seller had allowed all units to become vacant and allowed his insurance policy to lapse. In April, a buyer entered into a valid contract with the seller for the sale of the building. The parties agreed that the closing would occur on June 1. The contract complied with the Statute of Frauds but did not include a provision for the risk of loss. On May 20, the seller decided to have one last social gathering on the roof. After the party, the seller negligently left a grill burning on the roof. Flames from the grill caused a fire which eventually spread to consume the entire building. Upon hearing of the total destruction of the building, the buyer declared the contract void and refused to pay the purchase price. The seller seeks to enforce the contract despite the loss. There is no applicable statute. Who is likely to prevail? Answers: The seller, because title passed to the buyer under the doctrine of equitable conversion. The seller, because the buyer was in constructive possession of the building. The buyer, because the seller retains the risk of loss until the closing. The buyer, because the seller negligently caused the loss.

Answer choice D is correct. Absent an agreement or statute to the contrary, the doctrine of equitable conversion applies. Equitable conversion places the risk of loss between the execution of the contract and the closing date on the buyer, regardless of whether the buyer takes possession of the property. However, there is an exception when the loss is attributable to the intentional or negligent acts of the seller. In this case, the risk of loss would normally be on the buyer. However, the seller's negligent act in causing the fire is an exception to the doctrine of equitable conversion. Therefore, the buyer will prevail in voiding the contract. Answer choice A is incorrect because the seller's actions fall within an exception to the doctrine of equitable conversion. Answer choice B is incorrect because the doctrine and its exceptions apply regardless of the buyer's possession. Answer choice C is incorrect because it is the buyer who generally holds the risk of loss after the contract date.

Question 4217 A limited partnership purchased land with a loan from a bank. Neither the individual partners nor the limited partnership was personally obligated to repay the loan. As security for the loan, the limited partnership granted the bank a mortgage on the land. Subsequently, the limited partnership sold the land to a buyer. The buyer did not enter into an agreement with respect to the limited partnership's loan. After the sale, the limited partnership defaulted on the loan. The applicable jurisdiction follows the lien theory of mortgages. Can the bank foreclose on the land owned by the buyer? Answers: No, because the buyer did not agree to assume the limited partnership's loan. No, because no one was personally obligated to repay the loan. Yes, because the applicable jurisdiction follows the lien theory of mortgages. Yes, because the buyer took the land subject to the mortgage.

Answer choice D is correct. Although the buyer did not assume the limited partnership's loan, the buyer nevertheless took the land subject to the bank's mortgage. Consequently, although the buyer was not personally liable to repay the loan, the bank has the right to enforce its security interest through foreclosure. Answer choice A is incorrect because, even though the buyer did not assume personal liability for the limited partnership's loan, the buyer did take ownership of the land subject to the bank's mortgage. Answer choice B is incorrect because a mortgage is enforceable even though no one is personally liable to perform the obligation for which the mortgage serves as security. Answer choice C is incorrect because the fact that the applicable jurisdiction follows the lien theory of mortgages is irrelevant to the mortgagee's ability to foreclose on mortgaged property. Regardless of whether a jurisdiction follows the lien or title theory of mortgages, a person who acquires property that serves as security for an obligation takes the property subject to the loan.

Question 5898 A tenant leased a set of ten commercial storefronts spanning two city blocks. The lease was for a term of five years and complied with all relevant statutes. The lease was silent as to the effect of condemnation by the city. Three years into the lease, the city properly took one of the city blocks for public use pursuant to eminent domain and compensated the landlord accordingly. The city demolished five storefronts and began developing a public park. Upon this condemnation, the tenant stopped paying rent for all ten storefronts. In an appropriate action, the landlord sued the tenant for the unpaid rent on all ten storefronts. Is the landlord likely to succeed? Answers: No, because the lease terminated upon condemnation of some of the leased property. No, because the landlord breached the implied covenant of quiet enjoyment. Yes for the full amount, because the obligation under the lease is still in effect. Yes for half the amount, because the tenant is entitled to compensation.

Answer choice D is correct. Condemnation is the taking of land for public use or because it is unfit for use. The right of a tenant upon condemnation depends upon whether the condemnation is partial or complete. If the condemnation is partial, meaning only a portion of the leased property is taken, the tenant must continue to pay rent. The tenant is entitled to compensation for the portion that was taken. In this case, the condemnation was partial, as five of the tenant's ten storefronts were demolished. Due to the partial taking, the tenant is still required to pay rent according to the lease. However, the tenant must be compensated for losing half of his storefronts. Answer choice A is incorrect because a lease is terminated upon complete condemnation, not partial condemnation. Answer choice B is incorrect because the condemnation occurred through no fault of the landlord. The city properly exercised its eminent domain power and neither the landlord nor the tenant has the right to object. Answer choice C is incorrect. Although it correctly states that the landlord will succeed, the landlord has already been compensated for his loss by the city and cannot demand the tenant pay for property the landlord no longer owns. Thus, the tenant is not responsible for the full amount of the rent under the terms of the lease.

Question 4266 A man and woman were neighbors whose small yards were separated only by small bushes. After a discussion about building a one-foot thick stone wall to separate the two properties, the neighbors agreed that the man would pay for the wall, as the woman did not have the funds to do so, and as a consolation, the wall would be built on the woman's property so as to not reduce the square footage of the man's yard. Years later, the woman sold her property in a valid transaction with a buyer, but she told the buyer that the man had actually paid for and built the wall and that she had agreed to keep it there. Regardless, the buyer then spoke to the man about her desire to tear down the wall to open up the space and stated that she would pay for the destruction of the wall. The man objected to tearing down the wall. May the man prevent the buyer from tearing down the wall? Answers: Yes, because he has a separate security interest in the materials used to build the wall. Yes, because he paid for materials and construction of the wall. No, because the wall constituted an easement in gross. No, because the buyer validly purchased the land from the woman.

Answer choice D is correct. Fixtures, or structures built on real property, become part of the realty. Therefore, the wall became part of the woman's land, which was then sold to the buyer. When the buyer bought the woman's property, the wall, as a fixture, was included in the sale—whether the man paid for its construction or not. Absent some other legal prohibition not mentioned in the facts, the buyer could unilaterally decide to tear down the wall. Answer choice A is incorrect because once the materials were incorporated into the wall, they became an integral part of the property, and the man was not subject to any security interest in the materials. Answer choice B is incorrect because although the man paid for the wall, he had no security interest and no right to object to the wall's destruction. Answer choice C is incorrect because the wall did not constitute an easement on the woman's property. While easements in gross are not tied to the land, that is, they remain tied to one person, the relevant person is the owner of the dominant estate (here, the man), not the owner of the servient estate (here, the woman). Therefore, even if the wall did constitute an easement, it would benefit the man, and the buyer would be prevented from tearing it down. However, the wall would be more accurately characterized as a fixture, and therefore the buyer is the new owner of the wall.

Question 1580 A woman died testate. In her will, she devised a farm she owned to her husband for life, remainder to her niece. Her will did not specify the duties of the husband and the niece with regard to maintenance and expenses related to the farm. The husband took sole possession of the farm, did not farm the land, and did not rent the land to a third person, although the fair rental value was substantial. For two years in a row after the woman died, the county assessor sent the tax bills to the niece, but the niece did not pay the bills, because she and the husband could not agree on who should pay them. Finally, the niece paid the taxes to avoid a tax foreclosure sale. The niece then sued the husband for reimbursement for the two years' worth of property taxes. There is no applicable statute. Is the niece likely to prevail? Answers: No, because remaindermen are solely responsible for the payment of property taxes. No, because the county assessor sent the bills to the niece. No, because the woman's will was silent on responsibility for payment of property taxes. Yes, because the niece paid an obligation that was the sole responsibility of the husband.

Answer choice D is correct. In the absence of a contrary direction in the document creating the life estate - in this case, the will - it is the duty of the life tenant to pay all general property taxes that accrue during the continuance of the life estate. The only limitation on this duty is that the life tenant has no duty to expend more than the income that can be generated from the land. Because the fair rental value of the farmland was substantial, this limitation does not apply. If the remainderman does pay any property taxes due during the life tenancy, he or she is entitled to a judgment against the life tenant for reimbursement. Answer choice A is incorrect for the reasons stated above. Answer choice B is incorrect because it is not relevant to whom the county assessor sent the tax bills. The husband is responsible for the property taxes because the fair rental value of the farmland was substantial. As previously explained, answer choice C is incorrect.

Question 6425 Eighty years ago, a woman conveyed land to her son for life, then to her son's widow for her life, then to her son's children. At the time of the conveyance, her son was only five years old. The woman died a few weeks later, devising her entire estate to a charity. Thirty years ago, the son married. The following year the son and his wife had a child, but his wife died shortly thereafter. The child died at the age of 25, leaving her entire estate by will to a church. A year after his child died, the son married a woman who was 50 years old. The couple had no children. The son died this year and was survived by his widow, to whom he willed his entire estate. In a jurisdiction applying the common-law Rule Against Perpetuities, who has a properly vested interest in the land? Answers: The charity named in the woman's will in fee simple, because the son's widow was not a life in being at the time of conveyance. The church named in the will of the son's child in fee simple, because the son's only child had a vested remainder interest that passed by the child's will to the church. The widow in fee simple, because she had a life estate in the land and was devised her husband's entire estate. The son's widow for life, with the remainder to the church named in the will of the son's child.

Answer choice D is correct. Pursuant to the original deed, the son's widow has a life estate in the land that became a present possessory interest upon the son's death. The remainder interest in the son's children became a vested interest in his only child upon the son's death. Because this interest did not require the child to be living at the time of the termination of either the son's life estate or the life estate of the son's widow, this interest passes pursuant to the child's will to the church named in the will of the son's child. Answer choice A is incorrect because the widow's life estate does not violate the Rule Against Perpetuities. If the son had a widow, that widow would be immediately identifiable upon his death. Therefore, this conveyance is valid, even if the wife was not born when this property interest was created. Answer choice B is incorrect because it ignores the widow's valid life estate. Answer choice C is incorrect. The Rule Against Perpetuities requires that the future interest either vest or fail to vest within the applicable time period of a life in being plus 21 years. If there is any possibility that it will not be known whether the interest will vest or fail within that period, then the Rule has not been satisfied. When the son died, the class of his children closed, and the remainder interest in the son's children vested at that time. Therefore, the remainder interest in the son's children became vested in the son's only child upon the son's death. Because that child devised her interest in the land to a church, the church now owns that remainder interest. In addition, the son's life estate interest in the land terminated upon his death. As a result, he had no interest in the land to devise to his wife at that time. NOTE: This is NOT the "unborn widow" scenario, where surviving the widow is a condition precedent. Rather, in this question, the interest in the son's children will vest prior to the death of the widow (i.e., at the son's death).

Question 6219 A grandmother had lived in her family's mansion for her entire life, but she decided to sell the property and move into a smaller home. The grandmother, desiring to keep the mansion in her family, sold the mansion to her grandson at a below-market price. The grandmother included a right of first refusal clause in the valid, written deed to her grandson. The clause stated that, in the event the grandson, his heirs, devisees, or assigns attempted to sell the property to a non-family member, the grandmother, her heirs, devisees, or assigns would have the opportunity to purchase the property before the transfer. One year after the grandson purchased the property, he was approached by a buyer who offered him twice the price he had paid his grandmother. The grandson readily accepted, and immediately sold the mansion to the buyer. The grandmother subsequently read about the sale in the local newspaper, and brought an action against the buyer to enforce her right of refusal. The jurisdiction adheres to the common law Rule Against Perpetuities. Which of the following doctrines will help determine whether the grandmother will be able to enforce the right of first refusal clause? Answers: Doctrine of Worthier Title The Statute of Frauds The Parol Evidence Rule The Rule Against Perpetuities

Answer choice D is correct. Rights of first refusal are subject to the Rule Against Perpetuities. For the grandmother to prevail, she must prove that the right of first refusal clause was valid. Under the Rule Against Perpetuities, specific future interests are valid only if they must vest or fail by the end of a life in being, plus 21 years. If this requirement is not met, then the clause is invalid, and the grandmother loses. Answer choice A is incorrect because the Doctrine of Worthier Title is a rule of construction that prevents remainders in the grantor's heirs. Answer choice B is incorrect because the facts make clear that the clause and the deed met the Statute of Frauds requirements. Answer choice C is incorrect because the grandmother wants the clause in the deed to control, so any previous or contemporaneous evidence will not be helpful.

Question 1575 A woman died, devising land that she owned in another state to her daughter, who was then 17 years old. A neighbor who owned the property immediately adjacent to the land wrongfully began to possess the land at that time. For 24 of the next 25 years, the neighbor planted and harvested crops on the land, hunted on it, and parked cars on it. However, in the sixth year after he first took possession of the land, the neighbor neither planted crops nor hunted nor parked cars on the land because he spent that entire year living in Europe. The neighbor built a small gardening shed on the land, but he never built a residence on it. When the daughter was 28, she was declared mentally incompetent and had a conservator appointed to oversee her affairs. Since then, she has continuously resided in a care facility. The applicable statute of limitations provides as follows: "An ejectment action shall be brought within 21 years after the cause of action accrues, but if the person entitled to bring the cause of action is under age 18 or mentally incompetent at the time the cause of action accrues, it may be brought by such person within 10 years after attaining age 18 or after the person becomes competent." If the daughter's conservator wins an ejectment action against the neighbor, what will be the most likely explanation? Answers: The daughter was age 17 when the neighbor first took possession of the land. Because the daughter is mentally incompetent, the statute of limitations has been tolled. The neighbor never built a residence on the land. The neighbor was not in continuous possession of the land for 21 years.

Answer choice D is correct. The time period to acquire title by adverse possession in this jurisdiction is a minimum of 21 years. The neighbor has not been in continuous adverse possession for the entire 21-year period required because the neighbor spent one year in Europe after the first five years of possession. Answer choice A is incorrect. Because the daughter was a minor when the adverse possession began, she would be entitled to the protection of the tolling statute if it would extend the time in which she could bring her ejectment action. The tolling statute would not assist her in this case, however, because her minority ended within one year, when she turned 18, and the extension period of 10 years would allow her only 11 years in which to bring her action, less than the minimum time period of 21 years provided in the first part of the statute. The daughter's subsequent mental disability has no effect on the running of the statute. Answer choice B is incorrect because the daughter's mental disability has no effect on the running of the statute because she was not mentally incompetent at the time the neighbor began adversely possessing the land; thus, there would have been no tolling of the statute. Answer choice C is incorrect. There is no requirement that a residence be built on occupied land to adversely possess it.

Question 5958 A buyer entered into a contract to purchase a house from its owner. The contract called for the buyer to make equal monthly installment payments over 10 years, during which time the owner was to retain title to the house and the buyer was granted the right to occupy the premises. Under the contract, once the buyer made all of the required payments, the owner was to transfer ownership of the house to the buyer. The contract contained an acceleration clause under which all future installment payments were to become due in the event of any failure to timely make a required installment payment, and a forfeiture clause, which stated that time was of the essence and permitted the owner to terminate upon the buyer's failure to timely make a required installment payment, regain possession of the house, and retain any payments already made by the buyer. After making timely payments for seven years, the buyer failed to make three monthly payments. In accord with his rights under the contract, the owner filed a summary ejectment action to evict the buyer from the house. The applicable jurisdiction treats an installment land contract as a mortgage, follows the lien theory of mortgages, and does not recognize a mortgagee's right of strict foreclosure. The buyer appeared at the summary ejectment proceeding and tendered the missed payments. Should the court award possession of the house to the owner? Answers: Yes, because the buyer failed to timely make required installment payments. Yes, because the owner did not utilize self-help but acted through the judicial system. No, because the buyer tendered the missed payments. No, because there has not been a foreclosure sale.

Answer choice D is correct. This agreement is an installment land contract. Because the applicable jurisdiction treats an installment land contract as a mortgage, follows the lien theory of mortgages, and does not recognize a mortgagee's right of strict foreclosure, the owner cannot regain possession of the residence until a foreclosure sale has been held. Answer choice A is incorrect because, although the buyer's failure to timely make the required installment payments does constitute a default, this failure does not give the owner the right to retake possession of the residence. As noted with regard to answer choice D, the owner has this right only after a foreclosure sale is held. Answer choice B is incorrect. Although the owner did not attempt to regain possession of the resident through self-help, but instead chose judicial means, because the applicable jurisdiction treats an installment land contract as a mortgage and follows the lien theory of mortgages, the owner is entitled to possession only after a foreclosure sale is held. Answer choice C is incorrect. Because the applicable jurisdiction treats an installment land contract as a mortgage, the buyer possesses the equity of redemption, under which the buyer can redeem the property until there has been a foreclosure sale. However, the buyer's tender of the three missed payments does not satisfy the buyer's contractual obligations. Due to the presence of an acceleration clause in the contract, the buyer, as a result of the default, is required to tender the total amount of the remaining installment payments.

Question 5877 Anticipating the death of his mother and needing money, the only child of a terminally ill widow represented himself as owner of the widow's residence to a couple. The couple paid the son $200,000 for the residence. Although the couple did not immediately move into the residence, they promptly recorded the warranty deed they received from the son in the land records for the county in which the residence was located. The couple was unaware of the mother's ownership of the residence, which was also reflected in those same land records. One week after the land sale, the mother died. Upon her death, the residence passed by the terms of the mother's will to her son. The son, claiming ownership of the residence, has moved into it. The son has offered to return the $200,000 to the couple and pay for any expenses they have incurred with regard to this matter. The recording act of the applicable jurisdiction reads: "No conveyance or mortgage of real property shall be good against subsequent purchasers for value and without notice unless the same be recorded according to law." Of the following, which provides the best argument for the couple that they hold title to the residence? Answers: The son's rights to the residence have been lost through ademption. The couple's ownership of the residence is protected by the recording act. The son's ownership of the residence vests automatically in the couple under the shelter rule. The residence belongs to the couple by application of the estoppel by deed doctrine.

Answer choice D is correct. Under the "estoppel by deed" doctrine, a grantor who conveys a real property interest by warranty deed before actually owning it is estopped from later denying the effectiveness of his deed. Consequently, when the grantor does acquire ownership of the land, the after-acquired title is transferred automatically to the prior grantee. In this case, because the widow's son purported to transfer the residence to the couple by warranty deed prior to owning the residence, his acquisition of ownership of the residence upon the death of his mother results in the automatic transfer of his after-acquired title to the couple. Answer choice A is incorrect because ademption is not relevant under these facts. Under the doctrine of ademption, a devise fails (or is "adeemed") because the testator no longer owns the property upon death. Here, at the time of her death, the mother owned the residence. Answer choice B is incorrect. The recording act of the applicable jurisdiction is a notice act. Since the mother's ownership of the residence was recorded at the time of the purported sale of the residence by the son, the couple is deemed to have constructive notice of that ownership. Consequently, they cannot claim protection under the recording act. Answer choice C is incorrect. Under the shelter rule, grantors who are protected by the recording act protect (or "shelter") their grantees who would otherwise be unprotected. Here, the son, the grantor, was not protected by the recording act. Thus, the shelter rule does not serve to protect the couple.

Question 6423 The owner of a home mortgaged the property to secure a preexisting obligation to a bank. The bank immediately recorded the mortgage. The owner subsequently entered into a valid written contract with a buyer to sell her home to the buyer. The contract provided for the transfer of marketable title, but did not refer to the mortgage. Two weeks later, the buyer lost his job and decided he could not afford to purchase the property. In investigating his options for avoiding liability under the contract, the buyer discovered the existence of the mortgage. Following this discovery, he told the owner that the title was encumbered and that he would not close. The owner responded by offering to fully pay off the mortgage at the closing, using the proceeds of the closing. Despite this offer, the buyer refused to go forward. The owner then brought an action against the buyer for specific performance. If the court rules in favor of the owner in this action, which of the following is the most likely reason? Answers: The owner is not required to provide marketable title under the doctrine of equitable conversion. The buyer decided not to close because he lost his job, not due to the presence of the mortgage. The bank's mortgage does not have priority because it is not a purchase-money mortgage. The owner's offer to fully pay off the mortgage at the closing would render title marketable.

Answer choice D is correct. Unless otherwise agreed, the seller is not required to deliver marketable title until the closing. With regard to an outstanding mortgage, the seller is permitted to apply the proceeds from the sale of the property to the mortgage obligation. If the sale proceeds exceed the amount of the outstanding mortgage, the seller, by doing so, can eliminate this title defect. Therefore, if the court finds for the owner, it is likely because the current encumbrance does not prevent the owner from paying the obligation on the mortgage at closing and thereby delivering an unencumbered title to the buyer. Answer choice A is incorrect because the doctrine of equitable conversion provides that the seller retains legal title to real property during the pendency of the land sales contract, and that equitable title passes to the buyer upon entering the contract. This doctrine has no effect on the seller's obligation to deliver marketable title, and would not support a finding for the owner. Answer choice B is incorrect because the seller's obligation to comply with the contractual requirement of marketable title is not excused by the buyer's motive for asserting that the title is unmarketable. Answer choice C is incorrect. As a non-purchase money mortgage, the bank's mortgage is still an encumbrance on the title, regardless of whether it has priority over any other possible interests.

Question 2009 A couple entered into a contract to purchase a house from the owner. The couple did not record the contract of sale. Prior to the execution of the contract, the owner incurred a debt to a creditor. Subsequent to the execution of the contract, the creditor obtained a judgment against the owner. Unaware of the contract of sale, the creditor recorded her judgment in the land records for the county in which the house was located, thereby giving the creditor a lien against property owned by the owner in the county. After the owner deeded the house to the couple and they recorded the deed, the creditor sought to execute the lien and levy on the house. The couple filed an action to enjoin the creditor from executing the lien. The applicable recording act reads: No conveyance or mortgage of real property shall be good against subsequent purchasers for value and without notice unless the same be recorded according to law. Who will prevail? Answers: The creditor, because she recorded her judgment prior the couple's recording of their deed and without notice of their purchase of the house. The creditor, because she had reduced her claim to judgment. The couple, because they were protected by the recording act as purchasers for value of the house. The couple, because the doctrine of equitable conversion protected their interest in the house from the judgment creditor.

Answer choice D is correct. Upon execution of the land sales contract, the couple became the equitable owners of the house; the owner merely held legal title which he was required to convey at closing to the couple. Consequently, the creditor's judgment lien, which was obtained after the contract was executed, was not enforceable against the house because the house no longer belonged to the owner. Answer choice A is incorrect because, although the creditor did record her interest in the property without notice of the sale of the property to the couple, the recording act does not protect a judgment creditor who is not deemed to be a purchaser for value. Answer choice B is incorrect because, while the creditor had reduced her claim to judgment, the judgment was against the owner. As such, the judgment was enforceable only against property owned by the owner. Once the contract was executed, the owner no longer was equitable owner of the house. Answer choice C is incorrect because the recording act does not protect the couple. They did not record the contract of sale and, although they did record the deed, they had constructive notice of the judgment lien because it had previously been recorded.

Question 3138 Two brothers owned a pasture as joint tenants with the right of survivorship. The older brother had one child, a daughter. The younger brother was a bachelor. Together the brothers deeded a 20 percent interest in the land to the older brother's daughter. Recently, the older brother gave his daughter an additional 10 percent interest in the land. Under a traditional joint-tenancy analysis, what are the current ownership interests in the land? Answers: The two brothers have a 70 percent interest, and the daughter has a 30 percent interest, each as joint tenants with the other two. The two brothers have a 70 percent interest as joint tenants, and the daughter has a 30 percent interest as a tenant in common with her father and uncle. The two brothers each have a 35 percent interest and the daughter has a 30 percent interest, each as a tenant in common with the other two. The younger brother has a 40 percent interest, the daughter has a 30 percent interest, and the older brother has a 30 percent interest, each as a tenant in common with the other two.

Answer choice D is correct. Upon the transfer by the brothers of a 20 percent interest, the brothers held an 80 percent interest as joint tenants and the daughter held a 20 percent interest as a tenant in common with her father and uncle. The transfer by the older brother of an additional 10 percent interest to his daughter severed the joint tenancy between him and his brother, equally dividing the 80 percent interest between them and leaving them as tenants in common. In addition, since this second transfer was done by the older brother, his interest, which was 40 percent, was reduced by 10 percent, leaving him with a 30 percent interest. The younger brother retained a 40 percent interest. By the second transfer, the daughter's 20 percent interest was increased by 10 percent to 30 percent. Answer choice A is incorrect because the daughter never had a joint tenancy with her father and uncle since she did not receive her interest at the same time as they did and her interest was not the same amount as theirs. In addition, the two brothers are not joint tenants; the transfer by the father to the daughter severed the joint tenancy between them and left them with unequal shares. Answer choice B is incorrect because the brothers are not joint tenants; the transfer by the father to the daughter severed the joint tenancy between them and left them with unequal shares. Answer choice C is incorrect because the older brother alone transferred an additional 10 percent interest to his daughter and thus his individual interest is reduced to reflect that.

Question 5954 A married couple bought a house to use as a residence. Their bank loan was secured by a mortgage on the house. The following year, the couple granted a second mortgage to a savings and loan association in exchange for a loan. The proceeds from this loan were used in the couple's business. Several years later, the couple defaulted on both loans. The couple offered their interest in the house to the bank by deed in lieu of foreclosure and the bank accepted; the bank did not reserve the right to foreclose. What effect does this transaction have on the savings and loan association's mortgage? Answers: As an interest with priority over the bank's mortgage, the savings and loan association's mortgage is unaffected. As a junior interest to the bank's mortgage, the savings and loan association's mortgage is completely eliminated. The savings and loan association cannot foreclose on its mortgage, but must look to the personal liability of the couple, now that the bank owns the house. The house remains subject to the savings and loan association's mortgage.

Answer choice D is correct. When the bank accepted the couple's deed in lieu of foreclosure without reserving the right to foreclose, it extinguished the bank's mortgage on the house. However, this transaction did not affect the savings and loan association's mortgage on the house. The bank took title to the house subject to the savings and loan association's mortgage. Answer choice A is incorrect because, under the "first in time, first in right" rule, the savings and loan association's mortgage was the junior mortgage on the house. Answer choice B is incorrect. Although the savings and loan association's mortgage was the junior mortgage on the house, it was not eliminated by the deed in lieu of foreclosure transaction between the bank and the couple. This transaction eliminated the bank's mortgage, but left the savings and loan association's mortgage unaffected. Note that had the bank foreclosed on the property instead of accepting the deed in lieu of foreclosure, the junior interest likely would have been eliminated. Answer choice C is incorrect because, despite the transfer of title to the house to the bank, the house remains subject to the savings and loan association's mortgage. The transfer of property on which there is a mortgage by the mortgagor does not eliminate the mortgage. While its mortgage remains on the property, the savings and loan can still foreclose.

Question 4384 A man owned a parcel of land. In his will, the man specifically devised the parcel to his son and all of his personal property to his daughter. After a quarrel with his son, the man decided to sell the land. The man entered into a valid contract for a friend to purchase the parcel. Prior to closing, the man was killed in a car accident. The jurisdiction has an anti-ademption statute. Can the friend compel the executor of the man's estate to convey the land to the friend? Answers: No, the parcel should pass to the man's son pursuant to the terms of the man's will. No, the parcel should pass to the man's daughter under the doctrine of equitable conversion. Yes, the friend can enforce the sale contract, and the proceeds belong to the man's daughter. Yes, the friend can enforce the sale contract, and the proceeds belong to the man's son.

Answer choice D is correct. When the seller of real property pursuant to a contract dies before the conveyance of the property, the buyer can compel the executor of the seller's estate to honor the contract and transfer the property to the buyer. At common law, the proceeds from the sale of the property under contract were treated as personal property that passed to the devisee of the seller-decedent's personal property. The devise of the real property itself was treated as having been adeemed. In a jurisdiction that has adopted an anti-ademption statute, however, the devisee of the seller-decedent's real property is entitled to the sale proceeds. Here, the friend can compel the executor of the man's estate to convey the parcel of land to the friend, and the executor should transfer the sale proceeds to the man's son pursuant to the terms of the man's will and in accordance with the anti-ademption statute. Answer choices A and B are incorrect because the friend, as buyer of the parcel, is entitled to enforce the sale contract and compel the transfer of the property. Answer choice C is incorrect because the anti-ademption statute provides for the proceeds to go to the son.


Kaugnay na mga set ng pag-aaral

Chapter 4 Building an E-commerce Presence T/F

View Set

Pharmacy Practice and the Law Exam 1

View Set

Fund Development for Non Profit Orgs

View Set

Preload vs. Afterload Heart Nursing Quiz (RegisteredNurseRN)

View Set